Family Medicine MedU Questions

¡Supera tus tareas y exámenes ahora con Quizwiz!

A 55-year-old male with no significant past medical history presents for a routine physical exam. He last saw a doctor five years ago. Social history is remarkable for a 35-pack-year tobacco history since the age of 20. He indicates that his wife and children have urged him to quit smoking for the last few months. When you ask him if he has considered quitting, he replies, "I just don't see what the big deal is!" Which stage of change best describes this patient at this time? A. Precontemplation B. Contemplation C. Preparation D. Action E. Maintenance

A has been selected by the expert. Based on this man's response, it appears he has not actively considered quitting smoking despite his family's concern. All stems refer to a different stage in the Transtheoretical stages of change model. Given that he has not actively contemplated quitting, the best stage to describe this patient at this time would be the Precontemplation stage and not any of the other responses.

A 55-year-old male with a past medical history that includes hypertension, Chronic Obstructive Pulmonary Disease (COPD), and hyperlipidemia presents to clinic as a new patient for a general physical exam. History reveals that he has been smoking a pack of cigarettes daily since age 20. He drinks two beers daily. He is intermittently noncompliant with his medications. Review of the state immunization database reveals that the only immunization he has received as an adult was a tetanus diphtheria shot administered 12 years ago. Which of the following vaccine combinations would be most appropriate for this patient? A. Influenza, Meningococcal, and Zoster B. Influenza, Pneumococcal, and Tda C. Influenza, Zoster, and Tda D. Meningococcal, Pneumococcal, and Tda E. Meningococcal, Pneumococcal, and Zoster

B has been selected by the expert. Because this man has a diagnosis of COPD and smokes cigarettes, both annual Influenza and Pneumococcal vaccination are indicated. Because his last tetanus immunization was over 10 years ago and because he has not had a booster pertussis shot as an adult, a one-time TdaP is recommended. At this time meningococcal vaccine is recommended for adolescents and young adults and not indicated for this patient. Zoster vaccine is recommended to all adults at age 60 or older.

Ms. Burton is a 45-year-old female who has never been to a primary care provider. She presents today to establish care and get her health in order. Her concerns today are: fatigue, weakness, numbness, insomnia, feeling sad at times, anhedonia, increased appetite, weight gain, dry skin, and increasing hair loss within the past month. Her vital signs are: Heart rate: 78 beats/minute Respiratory rate: 18 breaths/minute Oxygen saturation: 95% Blood pressure: 152/84 mmHg Weight: 325 lbs Body Mass Index: 41 kg/m2 Today, her physical exam is significant for thinning hair, poor dentition, a systolic murmur heard at the left upper sternal border, an obese abdomen, and bilateral knee stiffness and pain on range of motion exam. Remainder of the physical exam is within normal limits. Which laboratory tests or studies can be done to rule out medical causes of insomnia, fatigue, and depression? A. Chest-X Ray B. CBC, CMP, and TSH C. HgbA1c, lipid panel, urine microalbumin D. CT head without contrast E. MRI brain with contrast

Correct Answer: B CMP can be used to detect electrolyte, renal and hepatic problems. TSH can be used to rule out hypo- or hyperthyroidism. CBC can be helpful to detect anemia and vitamin deficiencies. In addition, ESR can be used to test for rheumatologic disease. An ECG should be done if the patient is using drugs that might alter cardiac conductivity, such as TCAs.

Mrs. Anderson is a 60-year-old white female who comes in to clinic as a walk in appointment. She is tearful and is carrying a box of tissues in her hand. She says she doesn't know why but she has been very sad of late. She reports trouble falling asleep and staying asleep. She used to be the head of her Bridge club, but quit two weeks ago and doesn't feel like going out anymore. She also says she has lost interest in walking her dog, and now just allows him to use the doggie door to let himself out. She also says she feels weak and fatigued and no longer has the energy to do her gardening or shopping. She spends most of her day on the sofa crying while watching TV. She also reports a greatly diminished appetite. She denies suicidal or homicidal ideation, but she does have a history of a previous suicide attempt following her divorce seven years ago for which she was hospitalized. A recent CBC, CMP, CXR, TSH, U/A and CT of the head were all within normal limits. How long do the above symptoms need to be present in order to make the diagnosis of Major Depressive Disorder? A. One week B. Two weeks C. Four weeks D. Five weeks E. Eight weeks

Correct Answer: B Depressed mood or anhedonia and at least five of the following eight criteria must have been present for two weeks or longer. (Mneumonic = SIG E CAPS) - Sleep: Insomnia or hypersomnia nearly every day - Interest (loss of): Anhedonia (loss of interest or enjoyment) in usual activities - Guilt: Feelings of worthlessness or excessive or inappropriate guilt (which may be delusional) nearly every day (not merely self-reproach or guilt about being sick) - Energy (decreased): Fatigue or loss of energy nearly every day - Concentration (decreased): Diminished ability to think or concentrate, or indecisiveness, nearly every day (either by subjective account or as observed by others) - Appetite (increased or decreased) - Psychomotor agitation or retardation nearly every day (observable by others, not merely subjective feelings of restlessness or being slowed down) - Suicidal ideation: Recurrent thoughts of death (not just fear of dying), recurrent suicidal ideation without a specific plan, or a suicide attempt or a specific plan for committing suicide

Mr. Jones is an 82-year-old white male who presents to the office for his six-month chronic disease visit. His diabetes and hypertension are controlled on his usual home medications. He reports that his wife died four weeks ago, and he is now experiencing insomnia most days of the week and fatigue and loss of energy nearly every day; is not showering or shaving; has diminished concentration while playing chess with his neighbor; and is also experiencing loss of appetite but no weight loss. He denies any suicidal ideation and has no previous suicide attempts. Mr. Jones says he often hears his wife's voice while going to bed. He says he goes to church to pray and this helps him. You are trying to determine if your patient's symptoms are normal bereavement or if you should diagnose and treat him for Major Depressive Disorder (MDD). Which feature of Mr. Jones' case would suggest MDD rather than a normal grief reaction? A. Insomnia B. Change in appetite C. Lack of attention to hygiene D. Hearing wife's voice E. Fatigue

Correct Answer: C The loss of a loved one can be a traumatic event and it is normal to experience a period of grief. This reaction can be difficult to distinguish from major depression with symptoms of sadness, fatigue, changes in appetite, sleep disruption, and decreased concentration. Since your patient is also exhibiting diminished attention to activities of daily living, this may indicate the patient has MDD, as prolonged functional impairment is rarely a part of the normal grieving process. Other features that differentiate MDD from bereavement include: - Guilt about things other than actions taken or not taken at the time of death; Thoughts of death other than feeling that he or she would be better off dead or should have died with the deceased person; - Morbid preoccupation with worthlessness; - Marked psychomotor retardation; - Prolonged and marked functional impairment; - Hallucinatory experiences other than hearing the voice of, or transiently seeing the image of, the deceased person.

Ms. Rogers is a 75-year-old female who was found unresponsive in her house by her neighbor who had come over to help clean her house. An empty unlabeled pill container was found next to her on the bathroom floor. She was rushed to the ER, stabilized and is now in ICU on a mechanical ventilator. Which of the following are true regarding suicide in the elderly? A. Elderly persons attempting suicide are more likely to be married and living with their spouse. B. Elderly persons attempting suicide usually report good sleeping habits. C. Suicidal behaviors increase with age, but rates of completed suicides dont. D. Approximately 75% of the elderly who commit suicide had visited a primary care physician within the preceding month, but their symptoms went unrecognized. E. Firearms are the most common means of suicide in the elderly.

Correct Answer: D The USPSTF recommends screening all adults for depression, but especially patients with chronic diseases like diabetes, as they are at high risk for depression. The PHQ-2 inquires about the frequency of depressed mood and anhedonia over the past 2 weeks. The purpose of the PHQ -2 is not to establish a final diagnosis, but rather to screen for depression as a "first-step" approach. Patients who screen positive should be evaluated by the PHQ-9 to determine whether they meet the criteria for depression. Another screening tool which can be used is the Geriatric Depression Scale - Short Form (GDS-SF) which includes a series of 15 questions. Specifically related to suicide in the geriatric population: Elderly persons attempting suicide are more likely to be WIDOW(ER)S, AND LIVE ALONE; Elderly persons attempting suicide have REDUCED sleep quality; Suicidal behaviors DO NOT increase with age, but rates of completed suicides DO. Drug overdose is the most common means of suicide in the elderly.

A 4-year-old boy who recently emigrated from eastern Europe presents with his mother to your general pediatrics clinic. His mother reports that he has a chronic nonproductive cough during the day and night, mild wheezing for one month and failure to gain weight (his weight has dropped from the 50th to the 10th percentile for his age). His mother denies any high fevers, rhinorrhea, or night sweats. Which of the following are the next best diagnostic tests? A. Chest x-ray and tuberculin skin test B. CT of nasal sinuses C. Spirometry, before and after bronchodilator therapy D. Chest x-ray and methacholine challenge E. None needed, patient likely has habitual cough

Correct answer: A A. CXR and tuberculin skin test (TST) is the best choice. Signs and symptoms of primary pulmonary tuberculosis are few to none. Toddlers may present with nonproductive cough, mild dyspnea, wheezing, and/or failure to thrive (defined as weight < 5th percentile or drop in two percentile curves for weight). In children, TB can present without systemic complaints (fever, night sweats, and anorexia), severe cough, and sputum production. Regarding diagnostic tests, the TST is a practical tool for diagnosing TB infections. All children with chronic cough (more than three weeks) should be evaluated with a chest x-ray, as other pathology-such as lung abscess or malignancy-can also be detected on CXR. B. Sinusitis is often preceded by a URI, with nasal congestion as a prominent feature, leading to nocturnal cough due to post-nasal drip. These symptoms are not seen in our patient. Furthermore, a diagnosis of sinusitis is made clinically, with CT scan obtained only in complicated cases or cases resistant to treatment. Complications include cavernous sinus thrombosis, meningitis, and epidural abscess. C. Spirometry (pulmonary function testing) before and after bronchodilator therapy is the most specific means of determining whether or not a child has reactive airways. Asthma is a very common diagnosis in pediatrics, and may present with cough that is worse at night and exacerbated by exercise and cold air. Patients with cough-variant asthma present with only cough, typically nonproductive. However, given this patient's failure to thrive, a more serious diagnosis such as TB must be considered. Also, a chest x-ray is needed in all children with chronic cough (more than three weeks). D. Although a chest x-ray is appropriate in all children with chronic cough, a methacholine challenge (for asthma) would be inappropriate in this scenario. Although asthma is a common diagnosis, given the patient's failure to thrive, a more serious diagnosis must be considered. Further, a methacholine challenge is reserved for cases in which asthma is suspected and spirometry is normal or near normal, and should be performed by trained individuals. E. Habitual cough is caused by habitual perpetuation of a cough that begins with a viral URI. Continued coughing further irritates the airway, leading to stronger stimulation to cough. The cough is typically very loud, short, dry, brassy, and spasmodic. This cough is unchanged by exercise or cold air, and classically resolves during sleep. Although the patient in this case has a dry cough, his failure to thrive points to a more serious diagnosis (e.g., TB). All children with chronic cough (persisting longer than three weeks) need a CXR.

A 19-year-old female in her 38th week of pregnancy goes into active labor. Shortly after birth her baby is noted to have a high-pitched cry, tremulousness, hypertonicity, and feeding difficulties. The baby is otherwise developmentally normal and the remainder of the physical exam also is normal. What is the drug the baby's mother likely used during her pregnancy? A. Heroin B. Alcohol C. Marijuana D. Cocaine E. Tobacco

Correct answer: A A. Heroin is the correct choice. Opiate use during pregnancy may result in several different symptoms, including CNS findings (irritability, hyperactivity, hypertonicity, incessant high-pitched cry, tremors, seizures), GI symptoms (vomiting, diarrhea, weight loss, poor feeding, incessant hunger, excessive salivation), and respiratory findings (including nasal stuffiness, sneezing, and yawning). B. Alcohol is incorrect. Fetal alcohol syndrome has a distinct pattern of facial abnormalities, growth deficiency, and CNS dysfunction. These infants may also exhibit other neurobehavioral deficits such as poor motor skills and hand-eye coordination and learning problems, such as difficulties with memory, attention, and judgment. C. Marijuana is incorrect. There is limited evidence for a withdrawal syndrome associated with marijuana use. D. Cocaine use during pregnancy is not typically associated with withdrawal symptoms. Cocaine has been linked to subtle deficits appreciated later in childhood, including deficits in cognitive performance, information processing, and attention to tasks. E. Tobacco is incorrect. Smoking is not associated with the withdrawal syndrome described above. Smoking has been linked in a dose-dependent manner with lower weight newborns at birth. There is a two-fold increase in low birth weight even in light smokers (< 10 cigarettes per day). Smoking during pregnancy also has been associated with subtle neurodevelopmental deficits in some exposed children.

A mother brings her 20-day-old male infant to your clinic for the child's first visit. You learn that the infant was born at home to a 28-year-old G1P1, and the infant has not yet received newborn screening. During your history, you learn that the infant has been vomiting 2 to 3 times per day, and the mother reports that her son seems fussier than her friends' infants. On exam, you note an eczematous rash and a musty odor to the infant's skin and urine. Which enzyme deficiency would you expect the infant to display? A. Phenylalanine hydroxylase B. Cystathionine synthase C. Sphingomyelinase D. Alpha-L-iduronidase E. Glucose-6-phosphatase

Correct answer: A A. This infant likely has phenylketonuria (PKU), an autosomal recessive disorder of amino acid metabolism caused by a deficiency in the enzyme phenylalanine hydroxylase. Affected infants are normally detected by newborn screening, but can present with vomiting, hypotonia, musty odor, developmental delay, and decreased pigmentation of the hair and eyes. The best developmental outcomes occur if a phenylalanine-restricted diet is initiated in infancy. B. A defect in cystathionine synthase occurs in homocystinuria, a disorder of amino acid metabolism. Homocystinuria is inherited in an autosomal recessive pattern. Individuals display Marfanoid body habitus, a hypercoaguable state, and possible developmental delay. The condition can be diagnosed by testing for increased methionine in a patient's urine or blood. C. A defect in sphingomyelinase occurs in Niemann-Pick disease, a lysosomal storage disease. Children present by six months of age with hepatomegaly, ataxia, seizures, and progressive neurologic degeneration. Fundoscopic exam reveals a "cherry-red" macula. D. A defect in alpha-L-iduronidase occurs in Hurler syndrome, a type of autosomal recessive lysosomal storage disease. Children typically do not display symptoms until one year of age. Symptoms include hepatosplenomegaly, coarse facial features, frontal bossing, corneal clouding, and developmental delay. Affected individuals typically do not live past fifteen years old. E. A defect in glucose-6-phosphatase occurs in Von Gierke's disease, a glycogen storage disease. Von Gierke's disease is inherited in an autosomal recessive pattern. Individuals present with hypoglycemia, hepatomegaly, and metabolic acidosis.

At a routine well-child visit, the frantic mother of your 4-year-old male patient states that she thinks her son has some developmental delays based on what she hears from other parents. Although he knows how to do such things as throw a ball and copy a circle, he cannot brush his teeth on his own, tie his shoes, or hop on one foot. According to the AAP's Bright Futures, which of the following are development milestones for typical 4-year-olds? A. Throw a ball overhand, ride tricycle, build tower of 6-8 cubes B. Hop on 1 foot, copy a cross, brush teeth C. Tie a knot, copy squares D. Mature pencil grasp, print some letters and numbers E. Skip, draw a person with 6 or more body parts

Correct answer: B A. This choice is incorrect. Throwing a ball overhand, riding a tricycle, building a tower of 6-8 cubes, and copying a circle are developmental milestones for 3-year-olds. B. This choice is correct. A a normally developing 4-year-old should be able to hop on 1 foot, copy a cross, pour/cut/mash their own food, and brush teeth. C. This choice is incorrect. These are milestones for 5-year-olds. D. This choice is incorrect. These are milestones for 5-year-olds. E. This choice is incorrect. These are milestones for 5-year-olds.

Mark is a 5-month-old male who is brought to the urgent care clinic with a three-day history of rhinorrhea and non-productive cough. When he was born he was large for gestational age, and his exam then was notable for macrocephaly, macroglossia, and hypospadias. On physical exam now his vitals signs are stable. He has copious nasal discharge, but his lungs are clear to auscultation. On abdominal exam, you palpate an abdominal mass on the right side just below the subcostal margin. It is 7 cm in diameter and does not cross the midline. The abdomen is soft and non-tender with active bowel sounds. What is the most likely cause of his mass? A. Wilms' tumor B. Teratoma C. Renal cell carcinoma D. Hepatoblastoma

Correct answer: A A. Wilms' tumor is commonly associated with Beckwith-Wiedemann syndrome, a genetic overgrowth syndrome. Other features that may be seen in children with this syndrome include omphalocele, hemihypertrophy, hypoglycemia, large for gestational age, and other dysmorphic features. B. Teratomas are congenital tumors that are present at birth. These benign tumors that are often identified incidentally, or may become symptomatic due to mass effect of the lesion within the abdominal cavity. The aggressiveness of the tumor depends on the degree of differentiation. C. Renal cell carcinomas are much more common in adulthood. Risk factors include cigarette smoking and obesity. D. While children with Beckwith-Wiedemann syndrome can have hepatoblastoma (in addition to other types of tumors), this is not the most common tumor in this genetic condition. Note that hepatoblastoma may also be associated with familial adenomatous polyposis.

A 27-year-old woman comes into your office because she heard from her friend about a vaccination against cervical cancer and would like one. She has no medical problems and has had a Mirena IUD for three years. She has an allergy to latex and penicillin. She began having sex at the age of 18 and is currently sexually active with one partner. She occasionally uses condoms. She smokes half a pack of cigarettes per day. Her mother had endometrial cancer several years ago and had a total hysterectomy. Why is the patient not a good candidate for the Gardasil vaccination? A. Age B. Sexual activity C. Mirena IUD D. Allergy to penicillin E. Family history of endometrial cancer F. Tobacco use

Correct answer: A Gardasil is a vaccination against HPV types 6, 11, 16, and 18 approved for females ages 9 to 26. Cervarix protects against serotypes 16, 18, 31 and 45 approved for ages 10 to 25. Both are a series of three shots and recommended for females ages 11-18, optimally before sexual debut or shortly thereafter. The patient above is too old to receive the vaccination. Though vaccination before sexual debut is preferred, sexual activity (choice b) is not a contraindication to Gardasil vaccination. The other choices are not contraindications to vaccinations.

An 8-year-old healthy obese African American male with no past medical history is found to have a blood pressure of 125/90 mmHg on all four extremities on routine evaluation during an office visit for well-child care. Review of symptoms is negative. A physical exam and screening bloodwork are performed. Both are normal, with the exception of his blood pressure and obesity. What is the most likely diagnosis? A. Primary hypertension B. Renal artery stenosis C. Coarctation of the aorta D. Pheochromocytoma E. Hyperthyroidism

Correct answer: A A. The sole physical finding is hypertension. Given the mild hypertension and the patient's age, symptoms are unlikely to be present. Other etiologies should be ruled out, but review of symptoms, physical examination, and laboratory studies do not suggest other etiologies. B. Patients with renal artery stenosis are largely asymptomatic, but as the kidney function deteriorates, they may experience edema and dyspnea. On physical examination, patients will have hypertension (as a consequence of elevated angiotensin II and aldosterone) and possibly abdominal bruits. For laboratory studies, the patient will have elevated serum angiotensin II and serum aldosterone, as the kidneys attempt to compensate for a perceived decrease in glomerular filtration rate. C. Assuming sufficient severity of the coarctation, symptoms include chest pain, cold lower extremities, dizziness, syncope, exercise intolerance, failure to thrive, poor growth, headache, and dyspnea, among others. Distal to the coarctation, pulses will be diminished. Blood pressure will be lower in the lower extremities compared to the upper extremities. Murmurs may also be heart on auscultation. Coarctation of the aorta may also be associated with other congenital heart defects (bicuspid aortic valve in 50% of patients) and chromosomal abnormalities (Turner syndrome). D. Symptoms include headache, diaphoresis, palpitations, tremor, nausea, weakness, anxiety, nervousness, irritability, and weight loss, among other symptoms. On physical examination, patients typically present with tachycardia and severely elevated blood pressure. For laboratory studies, the patient will have elevated urinary VMA, urinary HVA, urinary metanephrines, and serum glucose. Pheochromocytoma may be associated with various syndromes, such as multiple endocrine neoplasia and Von Hippel-Lindau disease. E. Symptoms include heat intolerance, frequent bowel movements, increased appetite, diaphoresis, nervousness, restlessness, weight loss, tremor, hair loss, and palpitations, among others. On examination, patients typically present with tachycardia, hypertension, hyperreflexia, and goiter. The patient will have elevated T3, T4, and serum glucose, and TSH may be depressed or elevated depending on the etiology.

A 33-year-old G1P0 female with a history of medically controlled seizures gives birth vaginally to a boy with IUGR at 38 weeks' gestation. The newborn is noted to have dysmorphic cranial features and his head circumference is 28.5 cm (< 5th percentile). What is another associated abnormality you might expect to see in this newborn? A. Hepatosplenomegaly B. Cardiac defects C. Absent red reflex D. Chorioretinitis E. Tremors

Correct answer: B A. Hepatosplenomegaly in newborns is seen in metabolic diseases, storage diseases, HIV vertical transmission, intrinsic liver disease, and in congenital infections. Maternal anticonvulsant use does not cause hepatosplenomegaly. B. The mother was on an anticonvulsant for her seizures. Taking anticonvulsants during pregnancy may lead to cardiac defects, dysmorphic craniofacial features, hypoplastic nails and distal phalanges, IUGR, and microcephaly. Mental retardation may be seen. A rare neonatal side effect is methemoglobinuria. C. The red reflex is the normal reddish-orange reflection of light from the eye's retina that is observed when using an ophthalmoscope. An absent red reflex in a newborn can be due to congenital cataracts or a retinoblastoma. D. Chorioretinitis in a newborn may be due to congenital toxoplasmosis and CMV infections. It is not caused by maternal anticonvulsant use. E. Tremors in a newborn can be due to maternal substance use. They would not result from maternal use of anticonvulsants.

An 11-year old boy presents to clinic with wheezing. Mom states that in the past he has used inhaled albuterol and it has helped with wheezing and shortness of breath. On further history you find out that the patient experiences shortness of breath three times a week and is awakened at night by these symptoms once a week. What is the most appropriate outpatient therapy? A. Only rescue inhaler PRN B. Low dose inhaled corticosteroids C. Medium dose inhaled corticosteroids and course of oral corticosteroids D. Medium dose inhaled corticosteroids, LABA, and course of oral corticosteroids E. Course of oral corticosteroids

Correct answer: B A. Rescue inhaler (a short-acting beta agonist) i PRN is incorrect because this treatment is indicated in patients with intermittent asthma and have symptoms fewer than two days a week or two nights a month B. Low dose inhaled corticosteroid is correct because this patient has mild persistent asthma. His symptoms occur 3-6 days/week and 3-4 nights/month. C. Medium dose inhaled corticosteroids with a course of oral corticosteroids is incorrect, because it would be indicated in a patient with moderate persistent asthma when symptoms occur daily and more than one night per week. D. Medium dose inhaled corticosteroids, LABA, and oral corticosteroids is incorrect because this patient does not have severe persistent asthma. E. A course of oral corticosteroids alone is incorrect. Asthma needs to be managed long term to prevent exacerbations. An inhaled corticosteroid is indicated.

A 19-year-old G1P0 presents in labor to the ED at 38 gestational weeks. On interview it is discovered that the patient had irregular prenatal care, drank a couple of beers every weekend, and smoked 4 cigarettes a day. She delivers a baby boy who is small for gestational age. On exam, it is noted the baby has microcephaly, a smooth philtrum, and a thin upper lip. What do you suspect caused these features in the baby? A. Tobacco exposure B. Alcohol exposure C. Congenital rubella D. Vertically transmitted HIV E. Congenital CMV infection

Correct answer: B A. This choice is incorrect. While tobacco exposure can cause infants to be small for gestational age they typically do not have any characteristic facial features. B. This choice is correct. Fetal alcohol syndrome has very characteristic facial features, including a smooth philtrum, thinning of the upper lip, and small palpebral fissures. C. This choice is incorrect. Congenital rubella presents with sensorineural deafness, eye abnormalities (retinopathy, cataracts), and patent ductus arteriosus. D. This choice is incorrect. Typically, vertically transmitted HIV does not lead to recognizable symptoms at birth. This diagnosis cannot be completely ruled out without lab testing. E. This choice is incorrect. Symptomatic congenital CMV infection presents with microcephaly, jaundice, hepatosplenomegaly, low birth weight, and petechiae at birth.

A 10-month-old asymptomatic infant presents with a RUQ mass. Work-up reveals a normocytic anemia, elevated urinary HVA/VMA, and a large heterogeneous mass with scant calcifications on CT. A bone marrow biopsy is performed. Which of the following histologic findings on bone marrow biopsy is most consistent with your suspected diagnosis? A. Sheets of lymphocytes with interspersed macrophages B. Small round blue cells with dense nuclei forming small rosettes C. Hypersegmented neutrophils D. Stacks of RBCs E. Enlarged cells with intranuclear inclusion bodies

Correct answer: B A. This is incorrect, as sheets of lymphocytes with interspersed macrophages are associated with Burkitt lymphoma. B. This is the correct response. In addition to neuroblastoma, other tumors associated with small blue cells include Ewing's sarcoma and medulloblastoma, both of which tumors are seen in children. C. Hypersegmented neutrophils are characteristic of megaloblastic anemia, a condition associated with a vitamin B12 and/or folate deficiency, not malignancy. D. Stacks of RBCs suggest rouleaux formation, a phenomenon seen in multiple myeloma, a condition not seen in young infants. E. This describes the classic "owl's eyes" seen in CMV and other viral infections.

A 34-year-old woman who has no past medical problems nor is currently taking any medications comes into your office because she noticed a tender lump in her left breast starting approximately one month ago. She is worried because she has an aunt who had breast cancer that was BRCA positive. Her periods have been regular since they started at the age of 12 and occur every 32 days. She is currently menstruating. She has three children aged 12, 9, and 4. On exam, her BMI is 32 and her other vital signs are stable. On breast exam, you note a mobile rubbery mass of approximately 1 x 1cm and with regular borders that is tender to palpation. You appreciate no axillary adenopathy. The rest of her physical exam is unremarkable. Of the information provided, which of the following places this patient at increased risk for breast cancer? A. Age B. Weight C. Parity history D. Family history of cancer E. Age of menarche

Correct answer: B With a BMI of 32, obesity is the one risk factor for this patient based on the information given. Other risk factors for breast cancer include family history of breast cancer in a first degree relative (mother or sister - not aunt), prolonged estrogen exposure (menarche before age 12, menopause after 45, advanced age at first pregnancy), genetic predisposition (BRCA 1 or 2 mutation), advanced age (breast cancer risk increases with age, and this patient is relatively young), female sex, increased breast density and certain exposures (diethylstilbestrol, hormone or radiation therapy, heavy smoking).

You are seeing a 36-month-old boy for his well-child visit. His parents are anxious about ensuring that his development is appropriate. He passed a hearing screen at birth and, other than a few colds, has been generally healthy. He has never been hospitalized or had any serious illness. He is able to run well, walk up stairs, and walk slowly down stairs. He uses more words than the parents are able to count, but can use them only in short, two or three-word sentences. His speech is understandable. He can draw a circle, but not a cross. Neurologic examination shows normal cranial nerves, normal sensitivity, normal motor reflexes, and no Babinski sign. Which of the following is the most appropriate next step in the management of this patient? A. Perform a brain-stem auditory evoked potential hearing screen B. Perform a screening exam for autism C. Reassure the parents that the boy's development appears normal D. Refer the child to a developmental specialist for comprehensive evaluation E. Refer the child to a specialist for evaluation of his delayed motor development

Correct answer: C A. A brain-stem auditory evoked potential hearing test (BAER) may be indicated in infants who fail to meet language milestones if they cannot cooperate with other more comprehensive testing. A 36-month-old should be able to cooperate with behavioral audiometry, so a BAER is not indicated. In addition, this child has no evidence of language delay and does not require referral at this point. B. Autism is an increasingly diagnosed cause of developmental delay, but this child is not delayed and no mention is given of any autistic features, such as a lack of symbolic play, repetitive movements, or poor sociability. C. The developmental milestones mentioned in the vignette are within the range of normal for a 36-month-old child. In the absence of any other evidence of significant impairment, there is no indication for referral at this point. D. If there are reasons for concern on developmental screening tests, a referral may be indicated. However, the developmental milestones mentioned in the vignette are within the range of normal for a 36-month-old child. E. This child's motor milestones are not delayed, and no referral is indicated.

Sammy is a healthy male child brought into your office by his mother for a well-child examination. As part of your evaluation you assess his developmental milestones. He is able to run, make a tower of 2 cubes, has 6 words in his vocabulary, and can remove his own garments. What would you estimate Sammy's age to be based upon his developmental milestones? A. 12 months B. 15 months C. 18 months D. 30 months E. 36 months

Correct answer: C A. At age one year, gross motor skills include pulling to stand, standing alone, and perhaps first steps. Fine motor skills including putting a block in a cup and banging 2 cubes held in hands. At this age a child should be able to imitate vocalizations/sounds and babble. The majority of children this age will know 1 or 2 words in addition to "mama" and "dada." Social-emotional milestones at age one year are waving bye-bye and playing pat-a-cake. Running, building towers of blocks, removing clothing, and a 6-word vocabulary are more advanced skills than a 12-month-old would be expected to have. B. At 15 months of age, a child should be able to stoop and recover and walk well, put a block in a cup, have a vocabulary of a few words, wave bye-bye, and drink from a cup. Running, building towers of blocks, removing clothing, and a 6-word vocabulary are more advanced skills than a 15-month-old would be expected to have. C. At 18 months, a child should be able to walk backward, and 50-90% of children can run at this stage. An 18-month-old should be able to scribble, build a tower of 2 cubes, have 3-6 words in her or his vocabulary, and be able to help in the house and remove garments. D. At 2 ½ years of age, kids can jump up and throw a ball overhand. They can build a tower of 6-8 cubes, point to 6 body parts, name 1 picture, put on clothing, and wash and dry their hands. Sammy is only able to build a tower of 2 cubes, can remove his clothing but does not yet put clothing on, and his vocabulary is limited to 6 words-leading us to believe he is not 2 ½ years old. E. At age 3, children can balance on each foot for 1 second, wiggle their thumbs, name 4 pictures, name 1 color, name a friend, and brush their teeth with help. Sammy's vocabulary is only 6 words, he is not able to name a friend, he is only able to stack 2 cubes, and he has just starting running, but is unable to balance on each foot for 1 second.

A 9-month old baby boy comes to the clinic for a well child visit. The child is at the 50th percentile for weight, length, and head circumference. He is reaching all developmental milestones appropriately. The mother has no concerns at this visit. The child has previously received the following vaccines: 3 doses of DTaP, 3 doses of Hib, 2 doses of HepB, 3 doses of RotaV, 2 doses of IPV and 3 doses of PCV13, and no influenza vaccines. Which vaccines should the child receive at today's visit? A. Influenza, Hep B, IPV, DTaP B. Influenza, IPV C. Influenza, Hep B, IPV D. Hep B, DTaP, IPV E. Hep B, IPV, and MMR

Correct answer: C A. Influenza, Hep B, IPV, DTaP is incorrect. All three doses of DTaP have been given. B. Influenza, IPV is incorrect. The patient needs the third Hep B shot. C. Influenza, Hep B, IPV is correct. The patient needs a third Hep B, a third IPV, and a yearly flu shot starting at 6 months of age. D. Hep B, DTaP, IPV is incorrect. All three doses of DTaP have been given and the patient now needs a yearly flu shot starting at 6 months of age. E. Hep B, IPV, and MMR is incorrect. The patient also needs a yearly flu shot starting at 6 months of age and MMR is not given before 12 months of age.

A 10-year-old male comes to the clinic with a chief complaint of progressive cough for two weeks that began gradually. His cough is described as productive and wet with whitish sputum. His mother denies throat pain, vomiting, and diarrhea in his review of systems. His mother reports that he has been febrile up to 101.5°F daily. She thinks he is fatigued and has not eaten well in the past week. On exam, there is air passage throughout all lung fields, with crackles in the lower right lung field, but no other abnormal sounds. What would you likely find in your workup? A. Response to inhaled beta-agonist B. Hyperinflation in one lung field C. Alevolar consolidation in the RLL D. Positive PCR for pertussis E. Fluffy bilateral infiltrates and a large heart on chest x-ray

Correct answer: C A. Response to an inhaled beta agonist is a good test for RAD or asthma. Asthma is diagnosed clinically, usually in a school-aged child, with a history of recurrent wheezing. Associated findings might atopic stigmata, such as allergic rhinitis, food allergy, and atopic dermatitis. Without details in the history, asthma is a less likely diagnosis in this setting, and in the absence of wheezing, asthma is even less likely. B. This would be consistent with a foreign body aspiration which could produce cough and fever (if bacterial superinfection occurs). This diagnosis is usually considered in younger children. Constitutional symptoms (fatigue, decreased eating) make pneumonia a better diagnosis in this clinical setting. C. Pneumonia is the most likely cause for his symptoms and a chest x-ray would be a great confirmation of your suspected diagnosis. Eliciting a complete history might reveal history of an upper respiratory infection. Localization of crackles (discontinuous inspiratory sounds) to one lobe makes pneumonia more likely. D. Pertussis can produce a lengthy cough illness, but is not associated with fever or lung findings. E. Pulmonary edema due to CHF is a symmetrical process and less likely to present with a unilateral lung finding. Pulmonary edema should be suspected with crackles, but this clinical setting leaves pulmonary edema low on our differential.

A 9-year-old male presents to your clinic with discoloration under his eyes, persistent cough, and skin rashes. He is found to have wheezing on physical exam and increased lung volume bilaterally on chest x-ray. He has struggled with these complaints over the past three years but recently his symptoms have gotten worse, affecting him every other day. He is afebrile. He is found to have wheezing on physical exam and increased lung volume bilaterally on chest x-ray. What would be the most appropriate treatment for him? A. Oral antibiotics B. Short-acting beta agonist PRN C. Short-acting beta agonist PRN with low-dose inhaled corticosteroid D. Short-acting beta agonist PRN with medium-dose inhaled corticosteroid E. Long-acting beta agonist

Correct answer: C A. The patient's presentation is more consistent with asthma than an infection. The patient has had these complaints for the last few years. His skin rashes and lower eyelid darkening are consistent with allergic processes (atopy), which are associated with asthma. Furthermore, increased lung volumes bilaterally and persistent cough without fever also suggest asthma, thus antibiotics would not be appropriate. B. Cough and wheezing that occur intermittently (< 2 days/week) are consistent with intermittent asthma, which is treated with short-acting beta agonist PRN. C. Persistent cough and wheezing that affect the patient every other day (3-4 days with symptoms/week) are consistent with mild persistent asthma, which is appropriately treated with short-acting beta agonist PRN and low dose inhaled corticosteroid. The swelling under the eyes (allergic "shiners") and skin rash are other signs of atopy, as mentioned above. D. Short-acting beta agonist PRN with medium dose inhaled corticosteroid is the preferred treatment for moderate or severe persistent asthma, which corresponds to daily symptoms or symptoms throughout the day. E. The use of a long-acting beta agonist is reserved for severe persistent asthma, which corresponds to symptoms throughout the day.

A 3-year-old boy presents for a follow-up visit after being diagnosed with iron deficiency anemia. He is currently receiving oral iron supplements, 2 mg/kg of elemental iron daily. He has a dietary history of eating mostly sweet, bland, low-texture foods. What strategies may be used to improve his diet? A. Continue bottle-feeding B. Encourage eating small amounts of food throughout the day (grazing) C. Gradually introduce new foods and slowly decrease his old favorites D. Bargain and cajole with the child E. No change is needed; bland, low-texture foods are optimal for a child this age

Correct answer: C A. This choice is incorrect because the child should stop bottle-feeding now to improve his diet. Children usually stop requesting the bottle a few days after it has been discarded. B. This choice is incorrect because the child should be encouraged to restrict eating to 3 meals and 2 snacks per day, instead of "grazing" throughout the day. C. This choice is correct, because gradually introducing new foods and slowly decreasing his old foods will likely ease the transition to healthier diet choices and encourage long-term adjustment. D. This choice is incorrect because bargaining and cajoling is unlikely to be effective. He should be presented with healthy options only, and dessert should not be used as an incentive for healthy eating. E. This choice is incorrect because the child's current diet is not optimal. He should be eating a varied diet with the recommended servings of fruits and vegetables per day.

A 3-year-old boy described by his mother as a picky eater comes in for a regularly scheduled well-child visit. His mother complains that he has had less energy than usual for the past few months. There is a high clinical suspicion he is anemic. Which of the following is most correct? A. The most cost-effective test to diagnose anemia is a CBC. B. Lead screening is never warranted since a 3-year-old is usually not mouthing objects. C. The most likely cause of anemia in the question is picky eating resulting in low iron intake, which would cause microcytic anemia. D. The most common cause of anemia in this situation is folate deficiency. E. If anemia is due to poor nutrition, restarting the bottle will help the child recover the most.

Correct answer: C A. This choice is incorrect. The most cost-effective method to diagnose anemia is a screening hemoglobin. B. This choice is incorrect. Although most 3-year-olds stop mouthing objects, they may have been exposed to lead in the past but were not symptomatic at the time. C. This is the correct choice. The most likely cause of anemia in the question is picky eating, which can result in insufficient iron intake. Low iron intake causes a microcytic anemia. A girl with menometrorrhagia would present with iron deficiency, and her MCV also would be indicative of microcytic anemia. D. This choice is incorrect. Iron deficiency is the most common cause of anemia in this scenario. Note that folate deficiency may be associated with a goat's milk diet. E. This choice is incorrect. It is best to stop the bottle by age one year. Solid foods provide more complete nutrition, including iron. An additional concern of prolonged bottle usage is the development of dental caries.

Jane is an 8-year-old girl who presents to your clinic for follow-up after being hospitalized for status asthmaticus. She has just completed a 10-day course of systemic steroids. Given her history of moderate persistent asthma, her outpatient regimen includes Advair, a combined steroid and bronchodilator. She was also diagnosed with ADHD one year ago and was started on Concerta, 18 gm PO once a day. Her BMI today is at the 83rd percentile for her age, and her blood pressure is at the 98th percentile for her age. What is the most likely cause of her stage I hypertension? A. Obesity B. The blood pressure cuff is too big C. Medications D. Renal insufficiency E. Neurofibromatosis 1

Correct answer: C A. While obesity is a risk factor for hypertension, Jane is not overweight. A child is considered overweight when his or her BMI is between the 85th and 95th percentile for age. Obesity is considered > 95th percentile. B. Blood pressure would be falsely decreased if the cuff was too big, and, inversely, falsely elevated if the BP cuff was too small. C. Both steroids and amphetamines can cause increases in blood pressure, especially when used in combination. Steroids increase blood pressure by mimicking endogenous cortisol and the sympathetic fight or flight response. Amphetamines mimic norepinephrine, stimulating alpha and beta adrenergic receptors, causing an overall increase in blood pressure. D. Renal insufficiency can be a cause of secondary hypertension, but is unlikely in this patient. Risk factors that warrant investigation of renal causes for hypertension include recurrent UTIs, umbilical arterial/venous lines placed while a child was in the ICU, and a family history of renal disease. E. NF-1 can be associated with hypertension as a result of vascular malformations that affect blood supply to the kidneys; however, this child does not present with any signs of NF-1 (café au lait macules, neurofibromas, optic gliomas, lisch nodules).

A 21-year-old female with no significant past medical history experienced an inversion-type injury to her right ankle while playing soccer a day prior to presentation. She remembers immediate pain and swelling but was able to weight bear and limp off the field. She has noticed some significant swelling which is mostly still present. She has been icing the ankle since the injury as her coach recommended. Pain is still present near the lateral malleolus. Physical exam reveals that the lateral right ankle is edematous with purplish bruising, bilateral pulses are good and the patient had good sensation and motor function in both feet about equally. Palpation of the posterior edge of the lateral malleolus elicits significant pain from the patient. What is the next best step in the management of this patient? A. Reassurance B. Immobilize with cast or splint C. X-ray imaging of right ankle D. Emergent fasciotomy E. Rest, ice, Ibuprofen, compression and elevation and re-evaluate in one week Submit

Correct answer: C Based on the Ottawa Ankle Rules, tenderness of the lower 6 cm of the posterior lateral malleolus may predict fracture and justifies X-ray imaging of the ankle. Tenderness of the lower anterior lateral malleolus, on the other hand, is very common in ligamentous injury such that x-rays are not warranted. Because of the possibility that a fracture exists, the other treatment options are not correct in this scenario.

A 63-year old woman comes into your office for her annual preventive exam. She has hypertension and type 2 diabetes. She is not sexually active and experienced menopause at the age of 52. Her blood pressure is 125/83 and her physical exam otherwise is within normal limits. You recommend influenza and zoster vaccination. Her last colonoscopy was 8 years ago and her last mammogram one year ago was normal. She has never had an abnormal Pap smear. At the age of 45 she had a total hysterectomy for fibroids. You tell her she does not require a Pap smear today because: A. She has never had an abnormal Pap smear B. She is not sexually active C. She had a total hysterectomy for fibroids D. She is 63 years old E. She experienced menopause more than 10 years ago

Correct answer: C The patient described above underwent a total hysterectomy (total removal of the uterus and cervix with or without oophorectomy) for benign reasons (fibroids). USPSTF guidelines recommend against continued cervical cancer screening in patients whose uterus has been removed for benign disease and evidence showed cytologic screening to be very low yield and poor evidence that screening to detect vaginal cancers improves health outcomes in women after hysterectomy for benign disease. Cervical cancer screening should begin at the age of 21 and women between the ages of 65 and 70 who have had three or more normal Pap tests in the past ten years may choose to stop cervical cancer screening. Not being sexually active; age 63; only having had normal PAP smears and years since menopause are not reasons to stop screening for cervical cancer.

A 41-year-old male with no significant past medical history is brought to the Emergency Department after falling to the ground in the middle of a pick-up basketball game with friends. He did not lose consciousness nor hit his head when he fell. As he landed on the ball of his foot after having taken a shot, he recalls hearing a popping sound followed by immediate pain in the posterior right ankle. On physical exam, the posterior right ankle is edematous and palpation is tender. He is unable to plantarflex his right foot. What is the most likely diagnosis of his current condition? A. Ankle ligament sprain B. Calcaneal fracture C. Achilles tendon rupture D. Ankle tendonitis E. Ankle arthritis

Correct answer: C This is a classical description of an acute rupture of the achilles tendon. Middle-aged males are more commonly affected than other groups. The mechanism does not describe inversion injury making an ankle ligament sprain less likely and there is no direct trauma making fracture less likely. There is no history of overuse or chronicity making arthritis and tendonitis less likely.

A 5-year-old girl comes into your office for a well-child visit. The mother says that child is overall very healthy, but she highlights "occasional colds" and recently more frequent temper tantrums. She does well in preschool, is toilet trained, and enjoys eating mostly pasta, bread, and milk. She lives with her mother and father in a home built in 1985. Lab studies were significant for a mild anemia with a hemoglobin of 10.0 g/dL. You note that her hemoglobin was in the normal range at her 3-year-old visit. Which of the following is the most likely cause of her anemia? A. Chronic blood loss B. Lead poisoning C. Chronic illness D. Iron deficiency E. Hemoglobinopathy

Correct answer: D A. This choice is incorrect. There was no complaint of melena, and the child is overall healthy. Chronic blood loss would therefore be unlikely. B. This choice is incorrect. Risk of lead poisoning is increased in patients who live in homes built before the 1950s. Additionally, this patient does not complain of other symptoms suggestive of lead poisoning: weight loss, lethargy, vomiting, and learning difficulties. C. This choice is incorrect. The patient has no significant past medical history, and the review of systems is unremarkable. Chronic illness is unlikely. D. This choice is correct. Given the patient's age and preference for pasta and milk, the most likely cause of anemia would be iron deficiency. Treatment would include oral iron supplementation and increased dietary iron intake. E. This choice is incorrect. If she had a hemoglobinopathy, she would have been expected to have been anemic at her previous visit as well.

A 4-year-old patient presents with several months of cough. Mom also reports a history of red skin patches, which are pruritic, and allergies to peanuts, eggs, and mangoes. Which of the following would be characteristic of the cough that this patient would present with? A. Does not awaken patient from sleep B. Paroxysmal C. Barking cough D. Worse at night E. Associated with crackles on exam

Correct answer: D A. This choice is incorrect. This patient has asthma, which commonly presents with symptoms awakening the patient from sleep. Habitual cough disappears at night. B. This choice is incorrect. Paroxysmal coughs are associated with bacterial infections such as pertussis, Chlamydia, or mycoplasma. Foreign bodies can also produce sudden onset of cough. History could help to determine if the latter is a cause. C. This choice is incorrect. Barking coughs are associated with croup or other forms of subglottic disease. Foreign bodies can also produce this type of cough. D. This choice is correct. Asthma frequently presents with nighttime exacerbations. The cough often presents with wheezing and is usually a dry cough. E. This choice is incorrect. Diseases associated with crackles usually have intrinsic pulmonary involvement. Crackles can be fine or coarse and usually represent alveolar or small airway conditions.

A 47-year-old woman comes into your office for a health care maintenance exam. She has hypertension and type 2 diabetes. She is not sexually active and has not yet experienced menopause. There is no family history of cancer. Her blood pressure is 118/78, her BMI is 34 and the remainder of her physical exam is within normal limits. Her vaccinations are up-to-date, she has a PAP smear today and will have labs drawn. According to USPSTF, which of the following is the best recommendation to give her concerning mammography? A. Should have started at age 40 and every year thereafter B. Should have started at age 40 and every 2 years thereafter C. Start at age 50 and every year thereafter D. Start at age 50 and every 2 years thereafter E. Screening mammography is not recommended at this time

Correct answer: D Mammography has a sensitivity of 60-90% for detecting breast cancer and decreases breast cancer mortality. According to the most recent USPSTF guidelines, routine mammography is not indicated for women younger than 50 years old unless they fall into a high-risk category such as women with a BRCA mutation. The USPSTF recommends biennial testing for women between the ages of 50-74 years of age. There is insufficient evidence to assess the benefits versus risk of screenings in women after the age of 75. Other groups such as the American Cancer Society (ACS) and American College of Obstetricians and Gynecologists (ACOG) recommend yearly mammograms starting at age 40, continuing as long as the woman is in good health.

A 20-year-old female who is a long-standing patient at this clinic with no significant past medical history presents with first-time onset of dysuria accompanied by frequency and urgency for the past day. She thinks that there is a strange odor to her urine but denies any hematuria. In addition, she feels mild lower abdominal discomfort but denies fevers, chills, nausea, vomiting, constipation, diarrhea, or costo-vertebral angle (CVA) pain. She reports no known allergies She has never been sexually active and has no vaginal discharge nor irritation. LMP was one week ago and was typical. She wonders if using a perfumed bubble bath for the first time a few days ago might have triggered her symptoms. On exam, her vital signs are stable; she has no CVA tenderness and mild suprapubic discomfort. Urinalysis of a mid-stream catch is within normal limits, and a pregnancy test on the same sample, performed despite her history, is negative. Which of the following reflects best management in this situation? A. Insist that a pelvic exam and cervical DNA probe be performed today. B. Await urine culture results before any treatment. C. Reassure that this is not a urinary tract infection. D. Trimethoprim/sulfamethoxazole (one tab twice daily for three days). E. Urine DNA probe

Correct answer: D There is very good evidence that common typical symptoms of urinary tract infection (UTI) (e.g. dysuria and frequency) with the absence of vaginal symptoms are highly predictive of UTI in young women who have no systemic symptoms. The constellation of typical symptoms outweighs a normal urinalysis. Therefore, based on the symptoms presented, the most reasonable treatment option is to treat empirically with a standard antibiotic for a short course. It is not unreasonable to send urine for culture but management does not need to wait till that result is available. While it is always prudent to consider the possibility of sexually transmitted infection in this age group, based on the information presented about an established patient, it would be inappropriate to insist on a pelvic exam or await results of a urine DNA probe in the face of such strong UTI symptoms.

A 22-year-old female with no significant past medical history experienced an inversion-type injury to her right ankle while playing volleyball. The ankle quickly became edematous, but she used ice and was able bear weight on the foot. When she comes to clinic two days later, there is mininmal edema, she has good motor function, and has normal sensation. She has tenderness at the anterior lower lateral malleolus but not inferiorly nor posteriorly. X-rays are not indicated. You recommend continued relative rest and also tell the patient to keep it elevated and ice it several times during the day to help with the pain and swelling. You inform the patient that immobilization and compression is good for the conservative management of her condition. What is the best compression device to use in this situation? A. Tape B. Compression stockings C. Elastic wrap D. Semi-rigid ankle support E. Solid cast

Correct answer: D This patient apparently has a ligamentous injury to her anterior talo-fibular ligament. In recommending the RICE mnemonic, a semi-rigid ankle support (like an Air Stirrup) provides protection from repeat inversion injury while allowing the patient to actively dorsi- and plantar-flex her foot, which aids recovery. Compression stockings and elastic wrap do not provide adequate support. A solid cast completely immobilizes the ankle and delays recovery, while the evidence supporting taping of the ankle is lacking

Billy, a 7-year-old boy, presents to the clinic with complaints of headaches and episodes of feeling sweaty and flushed. He also reports that at times he feels as if his heart is racing. Billy was full term, had an uncomplicated birth, and has been otherwise healthy until now. On exam his BP is 120/80 mmHg and is the same in his upper and lower extremities. His weight and height are in the 50th percentile for his age. What is a likely cause of Billy's hypertension? A. Coarctation of the aorta B. Renal vascular disease C. Renal insufficiency due to renal scarring D. Catecholamine excess E. Primary hypertension

Correct answer: D A. Coarctation of the aorta should be suspected in a child with elevated BP (usually > 99th percentile), little family history of HTN, and a discrepancy between upper and lower extremity BPs. Some children with coarctation of the aorta may go undetected until presenting with hypertension at a school-age visit. It is important to pay special attention to the femoral pulses and to document BP measurement in a lower extremity. Billy's BPs in his upper and lower extremities are the same, making this diagnosis less likely. B. Umbilical arterial or venous lines as neonate (most often in premies) can predispose a child to renal vascular disease. Billy had an uncomplicated birth and did not go the neonatal ICU, making it less likely that his hypertension is due to renal vascular disease secondary to an umbilical arterial or venous line as a neonate. C. Recurrent urinary tract infections in childhood are one of the leading causes of hypertension and renal insufficiency later in life due to renal scarring following infections. UTIs are more common in girls. Billy does not have a stated history of urinary tract infections, and his palpitations and flushing are not consistent with hypertension secondary to UTIs. D. Catecholamine excess (pheochromocytoma or neuroblastoma) should be suspected in a child who is hypertensive and has episodes of sudden sweating, flushing, or feels that his heart is racing. Billy is exhibiting these signs and a urine catecholamine testing would be appropriate in this case. E. Most hypertension in children over 6 years of age, and in adolescents, is due to primary HTN. Obesity is an important correlate. Billy's flushing and racing heart would not be fully explained by primary hypertension. Also, Billy is an appropriate weight, making this diagnosis less likely.

George is a 7-year-old boy frequently in trouble at school for being disruptive and inappropriately talkative in class, not following directions set by his teacher, and not working well with classmates during group activities. His mother relates that at home George is always on the go, sleeping only 6 to 7 hours a night. He does not follow her rules all the time either, including not doing his homework, and sometimes putting himself in danger by doing things she tells him not to do, such as running away unaccompanied. Which of the following is the most likely diagnosis? A. Bipolar mood disorder B. Anti-social personality disorder C. Conversion disorder D. ADHD E. Rett syndrome

Correct answer: D A. Depression may be responsible for the inattention this child exhibits in school. It is not uncommon for childhood depression to lead to bipolar disorder, in which hyperactivity and impulsivity comprise the manic phase of the disorder leading to a decreased need for sleep. However, the symptoms of depression and mania present in separate phases, not concurrently. The mnemonic commonly used for mania is DIGFAST (distractability, irresponsibility, grandiosity, flight of ideas, agitation, sleep decrease, talkativeness). The mnemonic for depression is SIGECAPS (sleep disturbance, loss of interest, guilt, energy loss, concentration impairment, appetite changes, psychomotor retardation, suicidal ideation). B. Symptoms of anti-social personality disorder include inability to conform to societal norms, disregard of the rights of others, and often criminality. These individuals often exhibit impulsiveness due to their lack of consideration of the consequences of their actions. Males are affected more than females. However, a diagnosis of anti-social personality disorder can be made only in individuals older than 15 years, earlier than which a diagnosis of conduct disorder is appropriate. C. Conversion disorder is a subtype of somatoform disorder. Somatoform disorder would be suspected if no cause could be identified for reported physical symptoms. The drive is unconscious on the part of the patient, and symptoms are not intentionally produced or faked. Conversion disorder is more common in adolescents and involves a sudden loss of sensory or motor functioning. When the patient consciously creates physical symptoms this is referred to as factitious disorder. D. ADHD is characterized by the triad of impulsivity, hyperactivity, and inattention. Other symptoms include motor impairment and emotional labiality. ADHD is typically diagnosed before the age of 7 but persists into adulthood. Intelligence is usually normal, but individuals with ADHD commonly perform more poorly academically than would be expected for their IQ. E. Rett syndrome is an X-linked pervasive developmental disorder seen only in females; affected males die in utero or at birth. The characteristic symptoms involve regression of language and development, intellectual disability, ataxia, and hand-wringing. This disorder is typically diagnosed earlier, at about age 1-4.

A 2-year-old girl is examined as an outpatient. While waiting for the pediatrician, her mother reads her a short book. When they are done, her mother asks her to take the book and return it to a bookshelf in the room. The child is not able to hold a pencil and cannot write her name. She can kick and throw a ball, but cannot jump in place. Which of the following best describes this child's development? A. Delayed language B. Delayed social skills C. Advanced fine motor skills D. Advanced gross motor skills E. Age-appropriate development

Correct answer: E A. Delayed language is incorrect: A 24-month-old child is expected to use pronouns inappropriately, but should be able to follow two-step commands such as taking a book and returning it to a location in the room. B. Delayed social skills is incorrect: At 24 months of age, children are able not only to listen to short stories, they also engage in parallel play. C. Advanced fine motor skills is incorrect: While a child can hold a pencil at 24 months, the grip is immature and the child imitates pencil strokes. Children can remove their pants and socks at this age, but need help to undress completely. D. Advanced gross motor is incorrect: Being able to jump in place is a 30-month-old milestone. Being able to throw a ball overhand is expected at 24 months of age. E. The child in this vignette is developmentally appropriate for her age.

An asymptomatic, healthy 9-month-old female is found to have a palpable RUQ mass on exam. After further imaging and lab studies, the mass is diagnosed as a neuroblastoma that has involvement in the bone marrow as well. The mother is worried about the prognosis. Which of the following is true about the prognosis of neuroblastoma in this child? A. Lymph node involvement is a poor prognostic factor B. Prognosis of neuroblastoma is predictable C. Children who are older than 12 months have a better prognosis than younger children D. Favorable histology does not play a role in prognosis E. Non-amplification of the n-myc gene is a favorable prognostic factor.

Correct answer: E A. Due to the effectiveness of chemotherapy, neuroblastomas with lymph node involvement are still considered favorable, especially in the setting of other favorable factors, such as young age and differentiating histology. Though distant metastasis is a significant poor prognostic factor, regional lymph nodes do not significantly affect the outcome. B. Neuroblastoma has a broad spectrum of clinical courses. Some tumors may spontaneously regress, some may mature to a benign type, and yet other tumors can be very aggressive with metastases. Age plays a role in the prognosis, as most infants have a good prognosis even with disseminated disease, while infants over 18 months of age do not do as well. C. In infants less than one year of age, neuroblastoma tumors may spontaneously regress. Stage 4S neuroblastoma is a special category that is reserved for infants less than 12 months who have resectable primary tumors and metastases to the liver, skin, and bone marrow. Overall survival is over 85 percent. D. Favorable histology is a good prognostic factor in neuroblastoma, and is based on the differentiation of the cells involved. E. Non-amplification of the n-myc gene is one of the favorable genetics in neuroblastoma.

An 8-year-old boy is brought to clinic by his parents because they are concerned that he has not been doing his homework. His teacher recently called the parents to say that their son seems distracted in class, constantly interrupts other children when they are speaking, and is very fidgety. When you speak with the boy, he tells you that he did not know about the homework assignments and that he tries hard to pay attention in class. What is the next best step in management? A. Prescribe a stimulant medication for ADHD B. Suggest behavior modification for the child and parenting classes C. Group therapy for the child D. Do nothing, as this child's behavior is normal E. Contact the teacher to find out more about his behavior. Find out more about the child's behavior at home

Correct answer: E A. Pharmacotherapy is often used in combination with behavioral modification/group therapy for children diagnosed with ADHD. However, to be diagnosed with ADHD, one must have 6 or more symptoms in 2 or more settings for at least 6 months, and several of these symptoms must be present before the age of 12. The symptoms fall within the three categories of inattention, hyperactivity, and impulsivity. B. Behavior modification for the child and parenting classes for the parents are both used as treatment modalities in ADHD. Parents should be counseled on positive reinforcement, firm non-punitive limit setting, and how to reduce external stimuli. However, this child first requires further evaluation. C. Children with ADHD often learn best in group therapy, particularly social skills and self-esteem. Again, this child first requires further evaluation. D. Many school-aged children are easily distractible, impatient, and hyperactive. It is important to distinguish those who truly have ADHD from those who do not. Again, to have a diagnosis of ADHD, symptoms must be present in two or more settings. E. Contacting the teacher to find out more about the child's behavior at school and learning more about his behavior at home are the best ways to determine if 6 of the symptoms are present in 2 or more settings, which is required to make the diagnosis of ADHD. It also will be important to learn more about other aspects of this child's life, as there are several factors that can lead to acting out (including learning disability, hearing disability, family stress, and abuse).

A 64-year-old woman who is overweight with well-controlled hypertension comes to your office complaining of a lump in her breast that she noticed while showering. She denies any pain, tenderness, or skin changes. A pertinent review of systems is negative. Menarche began at the age of 10. Her first child was born when she was 29 and she had her second and last child at the age of 33. She experienced menopause at the age of 44. Her mother died of colon cancer when she was 65 and her father passed away from metastatic prostate cancer at the age of 70. She has no history of tobacco use ever and occasionally drinks a glass of wine with dinner. Her BMI is 34. Which of the information provided thus far puts the patient at decreased risk for breast cancer? A. Age B. Weight C. Age at first birth D. Age at menarche E. Age at menopause

Correct answer: E The patient experienced menopause at the age of 44, which shortens her time of estrogen exposure, a known risk factor for the development of breast cancer. Factors associated with decreased breast cancer risk include pregnancy at an early age, late menarche, early menopause, high parity and medications such as selective estrogen receptor modulators along with NSAIDs and aspirin. Risk factors for breast cancer include family history of breast cancer in a first degree relative (mother or sister), prolonged estrogen exposure (menarche before age 12, menopause after 45, advanced age at pregnancy, obesity after menopause), female sex, genetic predisposition (BRCA 1 or 2 mutation), advanced age (breast cancer risk increases with age), increased breast density and exposures (diethylstilbestrol, hormone or radiation therapy, smoking).

A 19-year-old female with no significant past medical history is involved as the driver in a motor vehicle accident and brought to the Emergency Department by EMS. She is complaining of severe pain in her right lower extremity that has been worsening since the accident. In addition, she has started to notice what she describes as "burning and tingling" in her right foot. On physical exam, her right calf is edematous and tender with tense overlying skin. There is no swelling or tenderness of the right foot or ankle but the right dorsalis pedis and posterior tibial artery pulses are barely palpable. She cannot confirm light touch of the foot and cannot wiggle her toes on command. What is the next best step in the management of this patient? A. Reassurance and icepacks q 2 hours B. Immobilize leg and ankle with a cast C. Urgent EMG of the right lower extremity D. Diagnostic imaging of right foot and ankle E. Emergent fasciotomy

Correct answer: E This clinical scenario describes acute compartment syndrome which is a vascular emergency. Emergent fasciotomy is the treatment of choice to relieve pressure in the calf and, if not performed, the limb could be lost due to acute ischemia. While emergent radiographs of the tibia and fibula are appropriate to evaluate for co-existent fracture, x-rays of the foot and ankle are not indicated. Reassurance, ice packs, urgent EMG and immobilization are all incorrect treatments and place the patient at risk of serious permanent adverse outcome.

A 55-year-old male comes to the clinic for a visit. He has read about the dangers of being overweight and inquires about which category he fits into. He is 5' 10'' (1.78 m) and weighs 220 lbs (100 kg), BMI = 31.6. Which of the following categories most accurately describes the patient based on his BMI? A. Underweight B. Ideal C. Overweight D. Obese E. Morbidly (very severely) obese

D has been selected by the expert. Based on BMI measurements, Underweight is considered < 18.5; Ideal: 18.5 to 25; Overweight 25 to 30; Obese 30 to 40; Morbidly (very severely) obese > 40.

A 55-year-old male with no significant past medical history and generally healthy behaviors presents to clinic for a health care maintenance exam. He says, "I'd like to get tested for all types of cancer." He does not have any family history of cancer. Review of systems is negative for any symptoms of prostate cancer, such as urinary frequency, urgency, retention, hematuria, weight loss, or back pain. Based on current US Preventive Services Task Force guidelines, which of the following is the most appropriate recommendation to give this man concerning screening for prostate cancer? A. PSA testing is recommended B. Referral for prostate biopsy under ultrasound is an option C. The benefits and risks of screening for prostate cancer are uncertain D. Recommend against PSA screening E. First, check a digital rectal examination

D has been selected by the expert. The US Preventive Services Task Force - at the time of printing, April 2014 - recommends against screening using PSA testing because the risks of harm of detecting and treating asymptomatic prostate cancer outweigh the known benefits. Digital rectal exam, prostate biopsy, and transrectal ultrasound are not recommended as screening tests either.

A 55-year-old white male with a family history of melanoma presents to the clinic for evaluation of a skin lesion on his back which appeared three months ago. His wife first alerted him to it, hasn't noticed it change and he has not noticed any symptoms associated with it. Physical examination reveals a 7 mm uniformly black macule that is symmetrically round with sharply demarcated borders on his upper back near the right shoulder. Which of the following characteristics would most justify it being biopsied today? A. Symmetry B. Borders C. Color D. Diameter E. Location

D has been selected by the expert. Using the ABCDE mnemonic, this nevus is not Asymmetrical, does not have irregular Borders, does not display Color variation and he does not describe any Evolution or change or symptoms. The only positive is that its Diameter is > 6 mm, which is considered a red flag supporting biopsy. Location is not considered a predictive factor for melanoma.

A 47-year-old caucasian female presents to the clinic complaining of an "itchy patch" on her skin. On further examination, you note a solid, elevated 1.5cm lesion on the extensor surface of the right forearm. How would you best describe the lesion? A. Plaque B. Papule C. Patch D. Nodule E. Macule F. Bulla G. Pustule

The correct answer is A. A plaque is a palpable flat lesion that is greater than 1cm in size. Papules are small raised palpable lesions that are less than 1cm in size. Nodules are solid raised palpable lesions which are larger in height and dimension than papules, occurring in the epidermis, subcutaneous tissue, or dermis. A macule is an area of skin discoloration that is 0.5-1cm in diameter, and a patch is a larger area of differently colored skin which is smooth to touch. A pustule is a small pus-filled raised lesion, while a bulla is a larger (greater than 1cm) fluid-filled raised lesion.

Ms. Marcos is a 65-year old Latina woman with a past medical history of Type 2 diabetes, hypertension, and hypercholesterolemia who presents with six months of insomnia despite self-medication with acetaminophen, diphenhydramine, and herbal remedies. She is 5' 2" and weighs 250 lbs. When considering a differential diagnosis, which one of the following is a common cause of insomnia in the elderly? A. Sleep Apnea B. Pneumonia C. Chronic sinusitis D. Asymptomatic coronary artery disease E. Hypoparathyroidism

The correct answer is A, sleep apnea. Sleep apnea occurs in 20% to 70% of elderly patients. Obstruction of breathing results in frequent arousal that the patient is typically not aware of; however, a bed partner or family member may report loud snoring or cessation of breathing during sleep. Some of the other most common causes of insomnia in the elderly are: - Environmental problems such as noise or uncomfortable bedding which are not conducive to sleep. - Drugs, Alcohol, and Caffeine such as over-the-counter, alternative, and certain recreational drugs. - Parasomnias such as restless leg syndrome/periodic leg movements/REM sleep behavior disorder. In restless leg syndrome, the patient experiences an irresistible urge to move the legs, often accompanied by uncomfortable sensations. In periodic leg movement and REM sleep behavior disorder, the patient experiences involuntary leg movements while falling asleep and during sleep respectively. - Disturbances in the sleep-wake cycle such as jet lag or shift work. - Psychiatric disorders such as primary depression and anxiety - Symptomatic cardiorespiratory disease (asthma, COPD, heart failure) - Pain or pruritus - Gastroesophageal reflux disease (GERD) due to heartburn, throat pain or breathing problems. - Hyperthyroidism The elderly frequently do not present with typical symptoms such tachycardia or weight loss, and therefore further laboratory studies may be required to detect this problem.

A 60-year-old Hispanic female presents to the office complaining of increased frequency of urination and fatigue for the past several months. She denies fevers, dysuria, back pain, diarrhea and abdominal pain. She has noted some weight loss without working on diet or exercise. Her past medical history is significant for hyperlipidemia and hypertension, for which she takes simvastatin and lisinopril. She is a non-smoker and consumes one to two glasses of wine per week. Her vitals are: Heart rate: 70 beats/minute Blood pressure: 130/70 mmHg Body Mass Index: 30 kg/m2 Physical examination reveals increased pigmentation in her axilla bilaterally. Her labs are as follows: Random blood glucose: 205 mg/dL Creatinine: 0.8 mg/dL TSH: 2.1 U/L. What test is needed to diagnose diabetes mellitus? A. The random blood glucose is sufficient B. Fasting blood glucose C. An oral glucose tolerance test D. HgbA1c

The correct answer is (A). Diabetes can be diagnosed with either an HbA1c > 6.5%, a fasting plasma glucose ≥ 126 mg/dl (7.0 mmol/l), a plasma glucose ≥ 200 mg/dL (11/1 mmol/l) two hours after a 75 g glucose load, or symptoms (such as polyuria, polydipsia, unexplained weight loss) and a random plasma glucose ≥ 200 mg/dL (11.1 mmol/l). Answers (B), (C) and (D) are incorrect, as the diagnosis of diabetes can be made based on random blood glucose with symptoms.

A 45-year-old female presents to your office complaining of left calf pain and swelling. She reports first noticing the pain after her yoga class earlier in the week. She denies having any chest pain or shortness of breath. She has no significant PMH. Her physical exam is significant for tenderness to palpation over the left calf. No edema or redness is present. The circumference of her left calf is 10 cm and the circumference of her right calf is 9 cm. What is the most appropriate next step in diagnosis? A. D-dimer B. MRI C. Lower extremity Doppler D. CT angiography

The correct answer is (A). This is the case where the pretest probability of a DVT is low, so that a negative D-dimer would save further testing. If the D-dimer is positive then the next step would be the lower extremity Doppler. CT and MRI are expensive and do not add anything to diagnostic accuracy in the majority of cases.

A 72-year-old woman with a 30-year history of Type 2 diabetes returns to your office for routine visit. She is taking 20 units of insulin glargine every morning and five units of insulin aspart with meals. The patient notes blurry vision for the past several months and a few days of dark spots in her vision. She denies headaches or nausea. What is true regarding diabetic retinopathy? A. 75% of people with diabetes only develop retinopathy 10 years after diagnosis. B. 40% of people with severe diabetes requiring insulin have retinopathy five years after diagnosis. C. Vision changes are an early sign of retinopathy. D. Primary care physicians should examine the retina on every visit for ongoing diabetes care.

The correct answer is (B). The patient's symptoms describe diabetic retinopathy. Proliferative retinopathy is prevalent in 25% of the diabetes population with ≥ 25 years of diabetes, but many patients have retinopathy much earlier. Early changes of retinopathy are asymptomatic. Patients need to see an ophthalmologist regularly for a dilated retina exam, not rely on a view of the retina from primary care physicians. Abnormalities seen include macular edema ( a common cause of blurry vision) and new blood vessel formation which can leak and cause dark spots in the vision. Diabetic eye disease often can be treated before vision loss occurs. Glaucoma (causing increased intraocular pressure) is 40% more likely in people with diabetes, but usually causes nausea, headaches, and narrowing of vision or halos around lights.

The patient in the prior vignette has made a plan to work on diet and exercise. Her A1c is found to be 8.9%. What is the best medicine to start at this time? A. A sulfonylurea B. Basal insulin C. Metformin D. Basal-bolus insulin

The correct answer is (C). After a trial of lifestyle changes, metformin is the first-line agent for treatment of Type 2 diabetes. Sulfanylureas are often used as second-line agents. Insulin is generally not used until two other oral medications are insufficient to control the blood sugar, but most people with Type 2 diabetes become insulinopenic over time and require insulin treatment.

A 61-year-old female has recently been diagnosed with Type 2 diabetes. Her fasting glucose was 240 mg/dL and her A1c was 8.9%. Her BP has been 148/90 and 146/86 at two separate office visits. Her home BP measurements have been in a similar range. Her creatinine is 0.9 and she has no known heart disease. She currently takes losartan 100 mg daily for a diagnosis of hypertension. Which of the following would be the most appropriate step in managing this patient's blood pressure? A. Make no changes to her medications as her blood pressure is at goal. B. Start lisinopril daily. C. Start amlodipine daily. D. Start metoprolol daily.

The correct answer is (C). Although the patient is over age 60, and the blood pressure goal is generally 150/90 mmHg, this patient has diabetes. JNC8 indicates a goal blood pressure of 140/90 mmHg for all patients with diabetes, regardless of age. While ACE Inhibitors have been historically the main first-line treatment for patients with diabetes, JNC8 does not make this specification, and allows the choice of any of the first-line treatments for hypertensive diabetic patients. These choices would include ACE inhibitors, angiotensin II receptor blockers (ARBs), thiazides, or calcium channel blockers. Beta blockers are not part of the management of hypertension, unless the patient has another indication (eg. CHF). In this case, lisinopril would not be an appropriate choice because the patient is taking an ARB (losartan) already. The combination of an ACE inhibitor and an ARB is contraindicated due to an increase in renal failure and hyperkalemia.

A 56-year-old Spanish-speaking male presents to your office for a six-month follow-up visit for DM Type 2 diagnosed at his previous visit with a HbA1C of 7.0%. At that visit, you discussed dietary modification including eliminating carbohydrates like rice from his diet. You also emphasized the importance of exercise and recommended he join a gym. On repeat testing of HbA1C today, you find that his HbA1C has increased to 7.5%. What might you do differently when counseling the patient at this visit? A. Suggest he get a personal trainer. B. Recommend he follow a more strict dietary protocol including elimination of meat from his diet. C. Find out from the patient whether barriers exist to his adherence to your previous suggestions. D. Tell him you are disappointed that he was not able to adhere to your previous recommendations.

The correct answer is (C). Before counseling a patient it is critical to assess barriers to adhering to the plan developed together. It is important to ask patients for understanding of the plan and whether they feel confident that they can follow the plan. Suggesting a personal trainer may be unrealistic for financial or time reasons. Eliminating meat may not appeal to him and telling him about your disappointment could make him less likely to tell you what is going on, or even making a follow-up visit.

A 30-year-old male with PMH significant for one month of progressive hoarseness and fever presents with a painful neck mass. He reports increased sweating, racing heart, diarrhea, and fatigue. His vital signs are: Temperature: 39.5 Celsius Heart rate: 85 beats/minute Respiratory rate: 19 breaths/minute Blood pressure: 130/70 mmHg On physical exam, you palpate a hard, fixed, painless nodule on the left side of the thyroid gland. Bloodwork results show: TSH: decreased Free T4: increased Which of the following is correct about thyroid nodules? A. 25% of patients with hyperthyroidism are caused by thyroid nodules B. The majority of thyroid nodules symptomatic C. 4% to 5% of thyroid nodules are cancerous D. Older patients with thyroid nodules usually have solitary nodules E. Thyroid radioactive iodine uptake and scan is the best initial test to evaluate a new thyroid nodule

The correct answer is (C). Four percent to 5% of thyroid nodules are cancerous. Only 5% cases of hyperthyroidism are due to nodules (A). Most thyroid nodules are asymptomatic and are found by physical exam or imaging done for another purpose (B). Older patients are more likely to have multinodular disease, and younger patients solitary nodules (D). The best initial imaging for a thyroid nodule is thyroid ultrasound (E). The patient in this case could either have an ultrasound or fine needle biopsy as this case is consistent with thyroid cancer, probably papillary.

A 65-year-old truck driver presents to your office complaining of right calf redness and swelling. He reports that he has had these symptoms off and on for some time but noticed that the redness and swelling on the right has worsened over the past few days and he also noticed fevers and chills. His PMH is significant for DM Type 2, COPD, and heart failure. He has a 25-pack-year smoking history. His physical exam shows a temperature of 101.1, 2+ pitting edema of his calves bilaterally, and dry, flaking skin over both calves. His right foot shows interdigital maceration and he has redness, warmth, and tenderness just inferior to the knee. When measuring the circumference of his right leg, you note that it is 12 cm compared to the left, which has a circumference of 10 cm. What is the most appropriate next step in this patient's care? A. Initiate treatment with Low molecular weight heparin. B. Order a lower extremity Doppler. C. Initiate treatment with antibiotics. D. Check a D-dimer.

The correct answer is (C). In light of the fever with redness and tenderness of the exam, the extra swelling is most likely due to cellulitis. There is no added benefit of treating with heparin and no particular reason to rule out a DVT with the Doppler or D-dimer.

A 55-year-old man presents to your office complaining of pain and stiffness in his knees. He used to play basketball three times a week with his friends but stopped about seven months ago because he was finding it difficult to keep up. His PMH is significant for hypertension, hyperlipidemia, and obesity. His BMI is 32. On exam, he is afebrile. There is no erythema or effusion of his knee joints but there is crepitus on movement of each joint. His range of motion of the knees is full. His x-rays show narrowed joint spaces and subchondral calcification. What is the most appropriate initial counseling for this patient? A. Instruct him to see an orthopedist for steroid injections. B. Refer him for knee replacement surgery. C. Develop a weight loss plan with the patient. D. Prescribe opioids for pain relief.

The correct answer is (C). Obesity is an important cause of mortality and morbidity. Knee pain is common in obese patients and losing weight can be helpful. Steroid injections generally help in osteoarthritis when there is joint inflammation, and this patient does not have signs of an effusion. There is no indication at this point for knee replacement surgery or opioids. Opioid pain medications are indicated for severe acute pain and in this case his pain is likely to be chronic.

A 65-year-old truck driver presents to your office complaining of right calf pain and swelling. He has recently returned from a four-day cross-country trip after which he had onset of his current symptoms. The patient reports that the week prior to his four-day trip, he was mostly in bed recovering from a bout of the flu. His PMH is significant for DM type 2 and COPD. He has a 25-pack-year smoking history. He denies having any chest pain or SOB. His physical exam shows 2+ pitting edema of his right leg. When measuring the circumference of his right leg, you note that it is 14 cm compared to the left, which has a circumference of 10 cm. What is the most appropriate next step in diagnosis? A. D-dimer B. MRI C. Lower extremity Doppler D. CT angiography

The correct answer is (C). Venous Doppler has the best sensitivity and specificity for diagnosing a DVT. D-dimer is best used when there is a low pre-test probability of a DVT. If it is negative then it is fairly certain that there is not a DVT. A positive test does not diagnose a DVT (poor specificity) but indicates the need for further testing. MRI and CT angiography are very expensive, carry risks with contrast, and do not add to the care unless an invasive intervention is being contemplated.

A 53-year-old man has been experiencing three months of weight loss and palpitations. Work-up for hyperthyroidism, which is high on your differential, is pending. Which of the following symptoms is inconsistent with a diagnosis of hyperthyroidism? A. Tremor B. Insomnia C. Gynecomastia D. Constipation E. Fatigue

The correct answer is (D) constipation. The other answer choices are typically associated with hyperthyroidism, while constipation is usually seen with hypothyroidism. Gynecomastia seen in 10% to 40% of patients with Graves disease and is thought to be because of sex hormone binding globulins being increased in Graves disease. Fatigue is common in both hyper- and hypothyroidism.

A 65-year-old male with known Type 2 diabetes mellitus presents to the Emergency Department with altered mental status. The patient experienced no known head trauma. His vitals are: Temperature: 38.1 Celsius Heart rate: 102 beats/minute Respiratory rate: 16 breaths/minute Blood pressure: 90/74 mmHg His mucous membranes appear very dry and he is started on IV fluids. Neurological exam reveals no focal deficits. His plasma glucose is found to be 700 mg/dL. Urinalysis reveals no ketone bodies. What is the most likely diagnosis? A. Thiamine deficiency B. Diabetic ketoacidosis (DKA) C. Cerebrovascular accident D. Hyperosmolar hyperglycemic state (HHS)

The correct answer is (D). HHS is seen typically in patients with Type 2 diabetes. It includes very high sugars > 600; ph > 6.4; dehydration; and lack of ketones in the urine and blood. Diabetic ketoacidosis is more common in Type 1 diabetes, and the patient will have ketone bodies in the urine. Thiamine deficiency can cause Korsakoff syndrome, and is typically seen in alcoholics with severe malnutrition, however, this patient is not a known alcoholic and doesn't appear malnourished. Despite the confusion in this patient, stroke is an unlikely diagnosis in this case given the lack of focal deficits on exam. Cardiac arrhythmia can cause dizziness, but is less likely to cause prolonged altered mental status.

A 35-year-old woman has been diagnosed with Graves disease and has decided to go for treatment with radioactive iodine (RAI). She has had good relief from taking propranolol for her palpitations. What is the best way to counsel this patient about what to do after her treatment? A. Tell her to stop her propranolol a week after she gets the RAI. B. Check her TSH level two weeks after her treatment with RAI. C. Describe the symptoms of hypothyroidism to her and tell her to expect to become symptomatic within one month. D. Check her TSH levels two to three months after her treatment and treat as necessary.

The correct answer is (D). The RAI takes several months to destroy the overactive thyroid cells. There is no point in checking the TSH after only two weeks and since she may still be symptomatic from her hyperthyroidism for several months there is no reason to stop her propranolol until she becomes euthyroid. It generally takes a number of months to notice symptoms of hypothyroidism, so while it is important to let her know what to expect in the future you do not want her to have unrealistic expectations.

A 24-year-old previously healthy female has been exhibiting some new concerning symptoms over the past four months. From her history, you gather that she's been having increased diarrhea, tremors, palpitations, and fatigue over this time. Your physical exam reveals a fine tremor in the bilateral upper extremities, sweaty palms, and a smooth, mildly enlarged thyroid gland. You determine that serum testing is warranted. Of the following lab results, which would best support the diagnosis of Graves disease? A. Low TSH, normal free T4, high free T3 B. Low TSH, normal free T4, normal free T3 C. Low TSH, low free T4, low radioactive iodine uptake, low thyroglobulin D. Low TSH, high free T4; high, diffuse radioactive iodine uptake E. Low TSH, high free T4; high, nodular radioactive iodine uptake F. High TSH, high free T4

The correct answer is (D). The diagnostic algorithm of the AAFP for hyperthyroidism can be found by clicking on the following weblink: http://www.aafp.org/afp/2005/0815/p623.html. As can be intuited from the answer choices, the first step is to check if serum TSH is low or high and if serum T4 is low or high. If TSH is low and T4 is high, primary hyperthyroidism is diagnosed, and a radioactive iodine uptake scan is next.

A 45-year-old woman presents to the ED with two weeks of abdominal pain, progressive weakness and palpitations. She notes an unexplained 3-lb weight loss as well as black, sticky diarrhea. Her vital signs are: Temperature: 99.1 Fahrenheit Heart rate: 117 beats/minute Respiratory rate: 22 breaths/minute Blood pressure: 92/67 mmHg She appears pale and diaphoretic. Her neck is supple and non-tender. Lungs are clear to auscultation bilaterally. Cardiac exam reveals elevated heart rate and a diastolic murmur. Her abdominal exam is notable for diffuse epigastric pain and hyperactive bowel sounds. Which of the following additional lab values might you expect? A. PaO2 of 60 B. Elevated LDH C. Free T4 of 15 mcg/dL (nml range 4.5-11.2 mcg/dL) D. TSH of 0.3 uU/mL (nml range 0.5-5.0 uU/mL E. Hgb of 8.4 g/dL (nml range 12.0-16.0 g/dL) F. Platelet count of 530,000 /mm3 (nml range 150,000-400,000/mm3)

The correct answer is (E). This woman appears to be suffering from anemia due to GI losses, as evidenced by her abdominal pain, weakness, tachycardia, diastolic murmur, and tar-like stools. There is some evidence of exam of hyperthyroidism (C), like tachycardia and diarrhea, but the other aspects of her presentation point to loss of blood. PaO2 is not impacted in blood loss (A), nor is platelet count (F). LDH is elevated in hemolytic anemia.

A 24-year-old male presents to clinic with history of upper abdominal pain, nausea and vomiting. He has previously had his gallbladder removed due to symptomatic gallstones. In reviewing his history, you want to screen for alcohol abuse due to the possibility of pancreatitis. Which one of the following is a sign of alcohol abuse: A. Failure to fulfill work, school or social obligations B. Denial of a drinking problem C. Positive answer to 1 of the CAGE questions D. 10 drinks per week E. 3 drinks per social occasion

The correct answer is A. Alcohol abuse is characterized by a maladaptive pattern of alcohol use with one or more of the following: failure to fulfill work, school or social obligations, recurrent substance use in physically hazardous situations, recurrent legal problems related to substance use, or continued use despite alcohol-related social or interpersonal problems. Screening for alcohol abuse and/or dependence can be done with modified CAGE questionnaire which consists of 4 questions. Have you ever felt: 1. The need to Cut down on drinking? 2. Annoyed with criticisms about your drinking? 3. Guilt about your drinking? 4. The need to drink an Eye opener in the morning? Positive answers to two or more of the above questions identify individuals who require a more intensive evaluation. Another screen developed by the American Society of Addiction Medicine (ASAM) is based on the number of drinks ingested per week. A positive screen for men is > 14 drink per week or > 4 drinks per occasion. A positive screen for women is > 7 drinks per week or > 3 drinks per occasion.

A 15-year-old female with a history of allergic rhinitis presents to the clinic with a five day history of productive cough and wheezing that is worse at night. She denies any shortness of breath, chest pain, or fever. The patient states that she has had similar symptoms in the past, especially when seasons change. The only reported past medical history is atopic dermatitis which is well-controlled with an over-the-counter steroid cream. What is the best next step? A. Prescribe a short acting beta agonist inhaler with a short course of oral steroids B. Prescribe a long acting beta agonist inhaler C. Prescribe a daily corticosteroid inhaler D. Prescribe a daily corticosteroid and long acting beta agonist inhaler E. Prescribe antibiotic therapy

The correct answer is A. Because the patient is symptomatic and wheezing, she needs to be treated with a medication to control her symptoms and treat her exacerbation. A short acting beta-agonist and short course of oral steroids do just that. After controlling her symptoms and treating her exacerbation, a more in-depth interview needs to be performed around the asthma symptoms and other ancillary tests need to be ordered and interpreted including a chest x-ray and pulmonary function test. Then, a decision can be made regarding daily controller inhalers (inhaled corticosteroids, chromolyn, etc). Finally, antibiotic therapy is not indicated in the treatment of an asthma exacerbation.

Which of the following is a comorbid condition that needs to be controlled in order to improve asthma symptoms? A. High BMI B. Insomnia C. Atopic dermatitis D. Anxiety E. Anemia

The correct answer is A. When a patient experiences difficulty with asthma control, the physician must consider and address comorbid conditions known to effect asthma control. These comorbid conditions include: gastroesophageal reflux (GERD), chronic sinusitis/uncontrolled allergic rhinitis, stress/depression, obstructive sleep apnea, and being overweight or obese. Successful treatment of these conditions often results in improved control of the patient's asthma symptoms.

Ms. Jones is a 35-year-old female with a significant past medical history of SLE who had been on NSAID therapy for the previous three months presented four weeks ago with heartburn. At that time, she reported episodic, mealtime epigastric burning radiating to the throat for the past few months. She has had no surgeries. Serologic testing for H. pylori IgG was reported to be positive a few days after her visit and she was begun on triple therapy. She now returns to the office for follow up. Today she denies any epigastric burning or tenderness. Physical exam is not significant. Which of the following is an accepted indication for performing repeat testing at this visit for H. pylori eradication? Choose the single best answer. A. Restarting of chronic NSAID therapy for SLE B. Documentation is required for all patients with confirmed H. pylori infection C. Treatment with triple rather than quadruple therapy D. Positive serologic test prior to therapy E. Age below 40 years

The correct answer is A. Indications for testing for proof of H. pylori eradication include: patients with an H. pylori-associated ulcer, persistent symptoms despite appropriate therapy for H. pylori, patients with H. pylori-associated MALT lymphoma, history of resection for early gastric cancer, and patients planning to resume chronic NSAID therapy. Documentation of eradication is NOT required for all patients who have tested positive.

A 23-year-old African American female comes to the clinic to discuss infertility. She states she was treated for an STD two years ago, and remembers receiving a Rocephin shot and taking a course of doxycycline. She admits to being hospitalized last year due to a severe infection with gonorrhea. She states that she and her fiance have attempted to have a baby for the past year, without success. She reports no dysmennorhea and has regular menses, but has pain during intercourse. Her last menstrual period was 4 weeks ago. Urine hCG is negative. Vital signs stable. Physical exam is unremarkable. The most likely cause of the patient's inability to conceive is: A. Pelvic inflammatory disease B. Uterine leiomyoma C. Endometriosis D. Candidiasis E. Ovarian cyst

The correct answer is A. The patient most likely was treated for pelvic inflammatory disease twice in the past. The most common cause of PID is Neisseria gonorrhoeae and/or Chlamydia trachomatis. It is treated with ceftriaxone and doxycycline to cover both bacteria. Pelvic inflammatory disease can cause infertility due to scarring of the fallopian tubes/ tubal adhesions. Uterine leiomyomas are benign neoplasms of the female genital tract, and are usually asymptomatic. They are not treated with antibiotics. They are unlikely to cause infertility, unless they are submucosal and distort the endometrial cavity of the uterus in a way that makes it an undesirable environment for pregnancy. Endometriosis involves functional endometrial tissue outside of the uterus, and often causes cyclical pelvic pain. Less commonly it may cause painful intercourse. Treatment may include surgery or a course of oral contraceptive pills to stop ovulation, or NSAIDs to treat symptoms. It can cause infertility, however is not the most likely cause of infertility in this patient due to her history. Candidiasis is caused by an overgrowth of yeast that is found naturally in a female's genital tract. The treatment is fluconazole. Candidiasis does not fit this patient's history, and is not a common cause of infertility. Ovarian cysts present with lower abdominal and pelvic pain. It may be severe pain if the cyst has ruptured. They are not treated with antibiotics making this less likely to fit the patient's history or be the cause of this patient's infertility.

A 63-year-old male with a past medical history significant for hypertension, COPD, and tobacco dependence is accompanied by his wife to a hospital follow-up clinic appointment. She is very concerned about her husband's recent hospitalization for a COPD exacerbation and asks what can be done to improve her husband's health. Which of the following holds the greatest long-term health benefit for our patient: A. Cessation of tobacco products B. Immunization against pneumococcus C. Prednisone taken daily D. Pulmonary rehabilitation program E. Spiriva inhaled daily

The correct answer is A; cessation of tobacco products. This is a key intervention in all patients with COPD who continue to smoke and can reduce the rate of FEV1 decline. The pneumococcal vaccine is recommended for COPD patients ≥ 65 years old or < 65 years old with FEV1 < 40% predicted and may reduce pneumonia. Long-term monotherapy with oral corticosteroids is not recommended. Pulmonary rehabilitation may improve dyspnea, walking distance, and quality of life but does not have as much supporting evidence as tobacco cessation. Spiriva is a medication for COPD, which may be used to decrease symptoms and/or complications, but no medication for COPD has been shown to modify long-term decline in lung function (DyneMed).

A 64-year-old African American man with a history of frontotemporal dementia is brought to the doctor for multiple dark spots found on the palms of his hands. The lesions are asymmetric and some have more than one color. Based on the information provided, which is the most likely finding on the biopsy? A. Nodular melanoma B. Acral lentiginous melanoma C. Squamous cell carcinoma D. Superficial spreading melanoma E. Benign nevus

The correct answer is B. Acral lentiginous melanoma is seen more often in dark-skinned people, and typically appears on the palms and soles of feet, including under the nails. Nodular melanoma presents as a single dark brown or black nevus on a sun-exposed area that grows deep into the skin. Superficial spreading melanoma presents as a nevus that has been growing and spreading along the skin surface. Benign nevi appear as small, symmetric, uniform colored moles. Squamous cell carcinoma tends to have a scaly, erythematous appearance more typical of a patch, plaque or nodule.

A 42-year-old female presents with nausea, vomiting and RUQ pain radiating to her back for one day. She reports a history of similar episodes but none have ever lasted for this long of a time period. Her vital signs are HR of 108, BP of 145/90, RR of 20, O2 saturation of 98% and temperature of 100.8F. Her labs reveal the following: WBC = 14.0 x 10^3/mL AST = 55 U/L ALT = 60 U/L Amylase = 70 U/L Lipase = 7 U/L What is the most likely diagnosis? A. Biliary colic B. Acute cholecystitis C. Acute pancreatitis D. Duodenal ulcer E. Hepatitis

The correct answer is B. Acute cholecystitis has similar pathophysiology to biliary colic but represents a stone that is lodged in the cystic duct. The symptoms are similar to biliary colic but typically last longer than 4-6 hours and may be more severe. Symptoms may include fever and elevated WBC. Biliary colic typically lasts 4-6 hours or less, radiates under right shoulder blade, often is accompanied by nausea, vomiting and can often follow a heavy, fatty meal. The hallmark of biliary colic is the stone is still mobile and gallbladder function resumes with relief of symptoms. Acute pancreatitis is often difficult to distinguish from biliary colic but will have an increased lipase and amylase. A patient with duodenal ulcer typically has epigastric pain that is relieved by food or antacids. Hepatitis is usually distinguished by malaise, anorexia, itching and icterus or jaundice. Signs of hepatitis would include hepatomegaly and elevated transaminases.

A 32-year-old female presents at your office for a preconception health visit. She is a G2P2, both deliveries were vaginal. Her first child was born with a neural tube defect. According to the USPSTF, what doseage of folate should this patient take daily before she gets pregnant? A. 1 mg B. 4 mg C. 400 mcg D. 800 mcg

The correct answer is B. US Preventative Services Task Force (USPSTF) recommends that all women planning or capable of becoming pregnant take a 400-800 mcg daily supplement of folate to prevent neural tube defects. The recommendation increases to 1 mg daily in patients with diabetes and epilepsy. In patients who have previously had a child with a neural tube defect, the recommendation increases to 4 mg daily.

A 52-year-old female presents for her third visit this year for productive cough. She has a 34-pack year history but has weaned down to only five cigarettes per day since she began to notice a cough. With her smoking history, you are concerned about the possibility of COPD. The gold standard for diagnosis of COPD shows: A. Brain natriuretic peptide >500 B. FEV1/FVC ratio of <70% of predicted C. Flattened diaphragm on lateral chest film D. Left ventricular function <40% E. Oxygen saturation level of <89%

The correct answer is B; FEV1/FVC ratio of <70% predicted. Spirometry (pulmonary function tests) is the gold standard for diagnosing COPD. If the FEV1 to FVC ratio is less than 70% of predicted (or less than the 5th percentile), then the patient has COPD. Brain natriuretic peptide levels >500 are suggestive of congestive heart failure. Flattened diaphragm on a lateral chest film may be suggestive of advanced COPD but is not diagnostic. Left ventricular function <40% is seen with systolic heart failure. Oxygen saturation levels <89% may be seen in those with COPD but is very nonspecific.

A 61-year-old male with a history significant for COPD presents to the emergency department for shortness of breath. Upon exam you see a thin male with perspiration on his forehead. He is having a difficult time answering questions because "he just can't catch his breath." You order an arterial blood gas on the patient. The results are pH 7.22 (7.34-7.44) PaCO2 81 mmHg (35-45 mmHg) PaO2 55 mmHg (75-100 mmHg) . The best next step is: A. Administer Rocephin B. Begin mechanical ventilation C. Immunize against influenza D. Provide nicotine replacement patches E. Repeat the test in 2 hours

The correct answer is B; to begin mechanical ventilation. This patient is in respiratory distress as evidenced by his physical exam of dyspnea and his ABG. He has a respiratory acidosis as his pH is low and he is retaining PaCO2. His PaO2 is also low. Initially you would want to improve his respiratory status with some type of mechanical ventilation such as nasal cannula, facemask, bipap, or even intubation if indicated by worsening of respiratory status. After stabilizing the patient, you might consider giving an antibiotic such as Rocephin if he was diagnosed with pneumonia or possibly a COPD exacerbation. Immunizing against the flu and providing tobacco cessation counseling are always good steps in patients with COPD, but would not be the initial step in this scenario. You might want to repeat the ABG after the patient is placed on mechanical ventilation to ensure improvement in his oxygenation status.

Mr. Rodriguez is a 32-year-old male who presents with three months of post-prandial epigastric burning. He reports no relief with antacid therapy. He has no allergies and no significant past medical history. He denies any hematemesis, odynophagia, dysphagia, hoarseness, or sore throat. Physical exam is unremarkable other than epigastric tenderness. H. pylori IgG serology is positive. Which of the following treatment regimens will most likely be successful in treating Mr. Rodriquez? Choose the single best answer. A. Omeprazole daily for 8 weeks B. Omeprazole twice daily for 4 weeks C. Omeprazole twice daily, clarithromycin 500mg twice daily, and amoxicillin 1g twice daily D. Omeprazole daily, clarithromycin 500mg twice daily, and tetracycline 500mg three times daily E. Omeprazole twice daily, tetracycline 500mg three times daily, and metronidazole 500mg twice daily

The correct answer is C The eradication of H. pylori required triple or quadruple therapy. Options include: twice daily dosing of a proton pump inhibitor (PPI) plus amoxicillin plus clarithromycin; a PPI once or twice daily (OR ranitidine twice daily) plus metronidazole, tetracycline and bismuth salicylate four times daily; or, for penicillin-allergic patients, twice daily dosing of a PPI plus clarithromycin plus metronidazole.

A 32-year-old female comes to the ER complaining of abdominal pain. She states the pain has been bothering her for the last week, and has progressively become worse. She denies nausea, vomiting, any episodes of diarrhea, or chest pain. She states she was diagnosed with high blood pressure six months ago, and is currently taking medication for it. She is sexually active with her boyfriend, and admits to having some pain during sex. She states she does not feel comfortable telling her boyfriend that she does not want to have sex. Her last menstrual period was two weeks ago. She denies a history of sexually transmitted infections, but says that she has not questioned her significant other because "he would be upset." Vital signs reveal a temperature of 98.7, blood pressure of 142/90, heart rate of 82, and respirations of 18. The patient is visibly uncomfortable during the physical exam, which shows a few bruises on her abdomen and lower back, which are at different stages of healing. Which of the following is an appropriate technique for screening for domestic violence? A. Request an appointment with the patients significant other, and asking him about abusing the patient B. Ask if she had ever been sexually abused as a child C. Ask the patient, "all couples disagree at some point in time. What happens when you and your partner argue or disagree?" D. Stand above the the patient while asking them questions E. Avoid direct questions about guns in the house or about drug/alcohol use by them or their partner

The correct answer is C To screen for domestic violence in a patient should involve asking indirect questions that include the opportunity to directly assess the safety of the the patient and their environment. Asking the patient's significant other to be present when screening for domestic violence could alter the answers of the victim, as well as damage the doctor-patient relationship. Sexual abuse in the past whether present or not, doesn't preclude the immediate issue of potential partner abuse/violence. Standing above the patient during the screening interview can intimidate the patient. Sitting down at the level of the patient allows for the intimidation to be less of a factor. It is appropriate to ask about drug and alcohol use, as well as the presence of guns in the victim's home environment in order to assess safety.

A 67-year-old female with an 80 pack year smoking history presents to an urgent care for worsening shortness of breath. She also reports her chronic cough is more frequent and is producing greenish mucous throughout the day. She is using her inhaler every two hours and has been taking an antibiotic left over from a previous sinus infection. On exam there is a whistling noise when she exhales. What other treatment should you consider at this time? A. Digoxin B. Furosemide C. Prednisone D. Propranolol E. Theophylline

The correct answer is C, prednisone. This patient meets the criteria for a COPD exacerbation as she has increased dyspnea, increased sputum volume and increased sputum purulence. Inhaled bronchodilators (albuterol) and oral glucocorticosteriods such as prednisone are effective treatments with or without an antibiotic. Digoxin is commonly used for patients with congestive heart failure or atrial fibrillation. Furosemide is a diuretic used for a number of things including volume overload. Propranolol is a beta-blocker used for hypertension and coronary artery disease. Theophylline is an oral methylxanthine that antagonizes adenosine receptors and increases cAMP. It can be used in asthma and COPD but would not be the initial choice of drug in this case.

A 23-year-old heterosexual Caucasian G0P0 female presents to your office with abdominal pain, fever and vaginal discharge. The discharge has an abnormal odor. She denies new or multiple sex partners. On pelvic exam, there is cervical motion tenderness and adnexal pain during the bimanual exam. A cervical culture is positive for chlamydia trachomatis. What risk factor for chlamyida infection is present in this patient? A. Sexual orientation B. Ethnicity C. Age D. Nulliparity

The correct answer is C. Chlamydia is the most common sexually transmitted bacterial infection in the U.S. Chlamydial infection is often asymptomatic and can result in serious complications-particularly for females. Risk factors for chlamydial infection include: age (women and men less than 24 years old and younger are at greatest risk), history of chlamydial or other sexually transmitted infection, new or multiple sex partners, inconsistent condom use, exchanging sex for drugs or money.

During your abdominal exam on the previous patient, you flex the patient's right hip to 90 degrees and take his right ankle in your right hand and with your left hand externally then internally rotate his hip by moving the knee back and forth. The patient denies any abdominal pain with this movement. What physical exam sign did you just perform and what disease is it ruling out? A. Psoas sign to rule out appendicitis B. Psoas sign to rule out cholecystitis C. Obturator sign to rule out appendicitis D. Murphy's sign to rule out appendicitis E. Obturator sign to rule out cholecystitis

The correct answer is C. During an abdominal exam, it is important to rule out other causes of abdominal pain. Appendicitis is another cause of acute abdominal pain and can be ruled out with a number of signs. One of these signs is the Obturator sign and the technique is discussed in the question stem. Another sign to rule out appendicitis is the psoas sign. This sign includes passive extension of patient's thigh as they lie on their side with their knees extend, or asking the patient to actively flex their thigh and hip. Pain with movement is often indicative of appendicitis due to inflammation and irritation of the psoas muscle. Murphy's sign is to identify cholecystitis. The technique involves having the patent breathe out slowly and completely, then gently placing your hands under the right costal margin. The patient is then instructed to take a deep breath in while you palpate for a hardened mass and determine if you elicit any significant tenderness.

A 25-year-old caucasian female presents with vaginal bleeding and cramping. Her last normal menstrual period was six weeks ago. The patient's vital signs are stable. On speculum exam of the vagina, there is no bleeding from the cervix. A quantitative beta-human chorionic gonadotropin (beta-hCG) level is 1492 mIU/ml. The patient is sent home and told to return to in 48 hours. Her beta-hCG on the return visit is 3000 mIU/ml. What is the appropriate next step in the management of this patient? A. Surgery for ectopic pregnancy B. Methotrexate for ectopic pregnancy C. Ultrasound to confirm intrauterine pregnancy D. Dilation and currettage Submit

The correct answer is C. Quantitative serum beta-hCG levels rise at a predictable rate, making it a useful tool to help distinguish between an intrauterine pregnancy and an ectopic pregnancy. During the first 6-7 weeks of gestation, beta-hCG levels double every 48 hours. With a beta-hCG level of 1500-1800 mIU/ml, a transvaginal ultrasound will be able to detect an intrauterine pregnancy. A transabdominal ultrasound will be able to detect pregnancy when beta-hCG levels > 5000 mIU/ml. An ectopic pregnancy will have beta-hCG levels lower than normal and levels aren't expected to double during early gestation. In the above case, the beta-hCG levels are doubling every 48 hours and a transvaginal ultrasound would be used to confirm intrauterine pregnancy.

A 62-year-old man with hypertension and diabetes is discharged home from the hospital, following a viral upper respiratory tract infection. He is prescribed several medications, including low dose aspirin. The patient asks you why he is taking aspirin along with the other medications. What is the reasoning behind your response? A. Aspirin helps decrease the risk of ischemic stroke in men. B. Aspirin helps decrease the risk of developing a hypercoagulable state. C. Aspirin helps decrease the risk of myocardial infarction in men. D. Aspirin decreases the risk of gastrointestinal hemorrhage.

The correct answer is C. The USPSTF recommends initiating aspirin therapy in men age 45 to 79 years to reduce the risk of myocardial infarction. In women age 55 to 79, the USPSTF advises taking aspirin to reduce the risk of ischemic stroke. For both men and women, the benefit of decreased risk from those outcomes must be weighed against an increased threat of gastrointestinal hemorrhage. Other groups disagree that all men be treated, and only use aspirin for high risk patients, where the benefits outweigh the risk of GI hemorrhage, which is increased in patients taking aspirin.

A 26-year-old professional football player comes to the clinic with the complaint of hair loss. On examination, the scalp is scaly, erythematous, and certain regions are purulent. There are several circular spots where the hair follicles are no longer present. KOH of skin shows hyphae. What is the most appropriate treatment for this patient? A. Punch biopsy of lesion B. Topical griseofulvin C. Oral griseofulvin D. Oral prednisone E. Topical prednisone

The correct answer is C. The description given in this scenario is suggestive of tinea capitis. Tinea capitis requires oral antifungals, rather than topical treatments. Topical therapies do not penetrate the infected hair shaft. Prednisone is not the treatment of choice for this condition. Biopsy is not necessary due to the typical appearance of this condition.

A 12-year-old boy is brought to the clinic with a linear maculopapular rash extending down both of his arms. He has just returned from a summer camp program in Wisconsin. Excoriation marks are visible. Which of the following is the most appropriate course of action? A. Biopsy the skin lesion B. Ask the patient to return to the clinic after 3 days C. Prescribe topical corticosteroids D. Prescribe oral antibiotics E. Prescribe oral corticosteroids

The correct answer is C. This child appears to have poison ivy. People affected by poison ivy tend to have linear lesions, due to contact with a branch or leaf or due to self-inoculation by scratching. The lesion are often fluid-filled, or vesicular, but over time can have less vesicular appearance. Treatment of choice is corticosteroids. For milder conditions, such as the case here, topical steroids are effective. For more severe cases in which a larger portion of the body is affected, oral steroids may be indicated. Antibiotics are not necessary, since this is not due to an infection. Biopsy is not necessary due to typical appearance and history. Treatment is important in order to reduce symptoms, so waiting 3 days would not be indicated.

A 34-year-old male comes to the clinic complaining of abdominal pain. He says the pain has been bothering him for the past two weeks. He reports episodes of diarrhea and constipation, with more episodes of constipation. He states he has noticed an increase in flatulence. He denies any nausea or vomiting. He has noticed mucus in his stools, but no blood. He states that he cannot recall if anything aggravates the pain, but admits to being under more stress than usual, due to his mother-in-law moving in with him and his wife. Vital signs show a blood pressure of 124/76, pulse of 74, respirations of 16, a temperature of 97.9, and oxygen saturation of 98% on room air. Physical exam is unremarkable. The most appropriate initial step in management is: A. Scheduling the patient for a colonoscopy to look for colon cancer B. Discussing the patients diet, and educating him about avoiding dairy products C. Performing a CBC, TSH, complete metabolic panel, and stool studies D. Prescribing an antispasmodic E. Scheduling the patient for a CT scan of the abdomen to rule out small bowel obstruction

The correct answer is C. The patient most likely has a diagnosis of irritable bowel syndrome. When a patient presents with abdominal pain, mucus in stool, episodes of diarrhea and/or constipation, it is appropriate to perform lab studies to rule out other etiologies, such as infection, autoimmune disease, or an obstruction. Since IBS is a diagnosis of exclusion, the diagnosis can be made once other sources of abdominal pain have been ruled out by appropriate studies. Scheduling a colonoscopy to search for colon cancer is inappropriate, as he is not presenting with bloody stool or significant weight loss. The patient is too young to undergo a screening colonoscopy without a history of a first degree relative who was diagnosed with colon cancer at an age 10 years older than his current age. Discussing the patient's diet is suitable, a trial of avoidance of dairy products would be dependent on history and a recommendation that includes this strategy may or may not be appropriate. Prescribing an antispasmodic would be appropriate after a diagnosis of IBS is made. Antispasmodics, such as Bentyl or Anaspaz, are pharmacological agents used to treat symptoms, and are not usually used long-term. Scheduling a CT scan of the abdomen to rule out a small bowel obstruction is inappropriate. The patient is not complaining of nausea or vomiting, and has no abdominal tenderness or hyperactive bowel sounds on physical exam, all of which would be suspicious for small bowel obstruction.

A 24-year-old female presents to the clinic complaining of a headache and arm pain. She is accompanied by her boyfriend, who insists on staying in the room with her during the visit. When asked to describe when she noticed the pain and how it occurred, the boyfriend states that the patient hit her head on the bathroom cabinet two days ago, and fell on her side afterwards, hitting her arm. The patient nods in agreement, while looking down at the floor. The next best step in obtaining an accurate history and physical exam is to: A. Direct all questions to the boyfriend, since it appears the patient does not want to discuss her situation with you B. Accuse the boyfriend of domestic abuse and call the police to report him C. Ask the boyfriend to leave the room so you can perform the physical exam, and explain that you always do that part of the exam with just the patient, and that he may join you after that part is completed D. Give the patient referrals to neurology and orthopedics to evaluate the patients headache and arm pain E. Prescribe Tylenol 650mg po BID to manage the pain, and tell the patient to return in a week if symptoms have not disappeared

The correct answer is C. When domestic violence is suspected and the partner is present for the visit, the best step in obtaining an accurate history and physical from the patient is to politely but firmly ask the partner to leave the room. To aid you in asking the partner to leave, it is suitable to tell them that the next part of the exam is done with the patient independently, and the partner may rejoin you and the patient after the exam is performed. If the partner still insists on staying, use other tactics, such as a test performed outside of the exam room or by taking the patient to get a urine sample. Directing questions to the boyfriend instead of the patient is inappropriate. You should always direct all questions to the patient. Accusing the boyfriend of domestic abuse and calling the police could precipitate the boyfriend acting out in the office towards you or the patient. It could also damage the physician-patient relationship, since it is not proven that the boyfriend is engaging in domestic violence towards the patient. Referring the patient to neurology and orthopedics is inappropriate. Initial workup for headache and arm pain can be done in the clinic. Referrals would be appropriate after the exam and workup are done, and not as an initial step to obtain a history and physical. Prescribing Tylenol would not be a step towards gathering a history and physical for the patient, and would be done at the end of the office visit.

increased frequency for the past week. She denies itching or pain in the vaginal area, but reports a burning sensation during urination that began a week ago. When asked if she has noticed blood in her urine, she admits to not paying attention to its color. She states that she has some abdominal pain in her pelvic area. She admits to being inconsistent with her birth control pills, and states she is sexually active. She does not recall the date of her last menstrual period. Urine hCG test is negative. Physical exam reveals suprapubic tenderness. She exhibits no costovertebral angle tenderness. Vital signs show a blood pressure of 126/78, temperature of 98.2, respirations of 18, heart rate of 82, and oxygen saturation of 97%. The most likely diagnosis is: A. Pyelonephritis B. Urosepsis C. Pregnancy D. Uncomplicated urinary tract infection E. Vaginitis

The correct answer is D A urinary tract infection presents with increased urinary frequency, dysuria, urinary urgency, and suprapubic pain. It can be associated with burning upon urination. Risk factors for UTIs include sexual intercourse, female gender, pregnancy, the presence of an indwelling urinary catheter, structural abnormalities (BPH, neurogenic bladder,etc.), immunocompromised states, diabetes, spinal cord injuries, and a history of recurrent UTIs. Pyelonephritis would present with fever, chills, and flank pain. It often is associated with urinary symptoms, such as increased frequency and dysuria. Nausea and vomiting may be present. Physical exam for a patient with pyelonephritis would reveal tachycardia, fever, and positive costovertebral angle tenderness. They may exhibit abdominal tenderness. The patient does not have fever, CVA tenderness, and is not complaining of symptoms that are indicative of pyelonephritis. Urosepsis would present with acute onset of fever, chills, tachycardia, tachypnea, and altered mental status. This patient does not have any of these presenting signs. Pregnancy does involve increased urinary frequency, and it a risk factor for UTIs. However, it is not a likely diagnosis in this case, as the patient's urine hCG is negative. Vaginitis would present with vaginal itching or irritation, as well as vaginal discharge. Examination of the discharge is necessary to discover the etiology. This patient is not complaining of any vaginal discharge or itching, making this a less likely diagnosis.

A 40-year-old male presents to the clinic with dry cough and wheezing for the past two days. He states that his symptoms began two days ago with a headache for which he took aspirin. He denies fever, but does report some continued shortness of breath. He is a smoker but only smokes 1-2 cigarettes a day for about six months. Physical exam is negative except for bilateral wheezing and erythema on the face. What is the most likely diagnosis? A. COPD B. Foreign body aspiration C. Pneumonia D. Asthma E. Pulmonary embolism

The correct answer is D. 21% of adults who have asthma have aspirin-induced asthma and should avoid aspirin and NSAIDS. Even though the patient is a smoker, he is less likely to have a COPD exacerbation because he has only smoked for six months. The patient's duration of symptoms (two days) and reporting no fever lead away from the diagnosis of pneumonia. Also, the clinical history gives you very little reason to suspect foreign body aspiration or pulmonary embolism. The diagnosis of asthma could be confirmed with pulmonary function testing.

Which of the following patients is an appropriate candidate for a exercise stress test? A. A 58-year-old male who presents to the emergency room with constant substernal chest pressure, diaphoresis and shortness of breath B. b. A 44-year-old female with a BMI of 40 kg/m2 and history of asthma C. A 48-year-old female with a history of intermittent anginal episodes that have been controlled on medications. D. A 52-year-old female with new atypical chest pain with a history of elevated cholesterol, smoking and family history of coronary artery disease

The correct answer is D. An exercise stress test is useful if the pretest probability of the disease is high as in D with 3 significant risk factors. False positives increase if the pretest probability is low so the usefulness of a stress test is low in B. Likewise if the diagnosis is certain as in A then there is not as much use in getting a stress, especially in an unstable patient. Since the purpose of invasive treatment is symptom control, the patient in C has controlled symptoms and hence no need for stress testing.

A 33-year-old G0P0 female presents to your clinic for her first prenatal visit. Her home pregnancy test was positive, she has been experiencing mild nausea for two weeks. Her last normal menstrual period was six weeks ago. Her initial prenatal visit should include: A. Quad screen B. Abdominal ultrasound C. Complete metabolic panel D. Complete blood count

The correct answer is D. An initial pregnancy evaluation should include CBC, RPR, HIV, Rubella, Blood type and Hepatitis B. A CBC is included to test for nutritional and congenital anemias as well as platelet disorders. The quad screen typically occurs later in the pregnancy (15-20 weeks) and transvaginal ultrasound (not abdominal ultrasound) is more reliable for dating a pregnancy at six weeks gestation. A complete metabolic profile is not a routine part of an intial pregnancy evaluation, and should be ordered only when indicated.

An 18-year-old non-smoking male comes to the clinic with cough, chest tightness, and difficulty breathing. His past medical history is positive only for allergic rhinitis as well as an undiagnosed chronic cough primarily at night since he was 4 years old. He has no past surgical history and no recent travel. On physical exam, you notice the patient appears in mild distress, has hunched shoulders, is using accessory muscles during respiration, and is only able to talk in two to three word sentences. His vitals are: Pulse 125/min, O2 sat 88%, BP 100/70 mmHg. On auscultation, you hear wheezing on inspiration and expiration throughout both lungs. You also notice a prolonged expiratory phase. What is the most likely diagnosis? A. Cystic Fibrosis B. CHF exacerbation C. Foreign body aspiration D. Severe asthma exacerbation E. Pulmonary embolism

The correct answer is D. Based on the clinical presentation including the patient's age and uncomplicated past medical history, the most likely diagnosis is an asthma exacerbation. The presence of hypoxemia places this in the "severe" category. While pulmonary embolism and foreign body aspiration are still on your differential, they are both much further down again based on the patient's age and reported past medical history. Neither cystic fibrosis nor heart failure are likely given the past medical history.

Which of the following symptoms are most likely to be due to acute coronary syndrome? A. 23-year-old male with acute onset of difficulty breathing and hyperresonance upon lung auscultation B. 42-year-old woman with a pulsating pain in the center of her chest at night C. 35-year-old man with chest pain radiating down his left arm after falling off a ladder at work one week ago D. 59-year-old woman with palpitations that increase with exercise and are associated with nausea and vomiting E. 55-year-old woman with diffuse central chest pain that is worse when lying down

The correct answer is D. Palpitations, nausea, and vomiting are seen as prodromal symptoms of ACS in women more than men. Young men with difficulty breathing and hyperresonance are likely to have a pneumothorax. Pulsating pain is one of the types of pain, along with pleuritic and positional pain, that is less likely to be related to heart disease. Chest pain in a young man associated with trauma is less likely to be ACS.

A 35 year old overweight woman in good health comes to the clinic for a routine physical. Which of the following screenings are recommended by the U.S. Preventative Services Task Force (USPSTF)? A. Complete blood count B. Thyroid stimulating hormone levels C. Cholesterol panel D. Blood pressure screening E. HbA1C

The correct answer is D. The only one of these that the USPSTF recommends in this age group is blood pressure screening. It is recommended to screen women with a cholesterol if there are risk factors for CHD.

A 28-year-old male comes to the Emergency Department with shortness of breath, cough and wheezing for the last 4 hours. He states that he was diagnosed with asthma recently, and is currently using inhaled corticosteroid with a long acting B2-agonist daily to control his symptoms. His RR is 34/min; Temp: 98.8 degrees F; O2sat: 88%; BP: 130/85 mmHg. What treatment should be given to this patient first? A. Albuterol breathing treatment using a nebulizer B. Intravenous corticosteroids C. Ipratropium breathing treatment using a nebulizer D. Oxygen E. Intravenous third generation cephalosporin

The correct answer is D. This patient is in acute respiratory distress with a decreased oxygen saturation and increased respiratory rate. The first treatment this patient should receive is oxygen supplementation. After giving the patient oxygen, you can then continue treatment for his acute asthma exacerbation with albuterol and ipratropium breathing treatments and intravenous or oral corticosteroids. Antibiotics are not indicated for treatment of asthma exacerbations.

A 24-year-old African American G1P0 at 14 weeks presents with vaginal bleeding and abdominal cramping. On examination her vital signs are: blood pressure 120/75 mmHg, pulse rate 74/minute, temperature 98.4 degrees F taken orally, respiratory rate 18/minute, and oxygen saturation 99% on room air. On pelvic examination, there is a small amount of blood in the vagina, the cervical os is open, and there is no cervical or adnexal tenderness noted. On pelvic ultrasound, an intrauterine gestational sac with a yolk sac is seen. What is her diagnosis? A. Complete abortion B. Threatened abortion C. Missed abortion D. Inevitable abortion

The correct answer is D. When the cervical os is open or dilated bleeding in early pregnancy is classified as an inevitable abortion. When the cervical os is closed with bleeding before 20 weeks, the correct diagnosis is a threatened abortion. A missed abortion is fetal demise without cervical dilation. Complete abortion occurs when products of conception have been completely expelled from the uterus.

Ms Smith is a 38-year-old female with a significant past medical history of hypercholesterolemia who presents to her primary care physician after 4 weeks of episodic epigastric discomfort. She reports a recurrent ache-like sensation in the epigastric region occurring about 10 minutes after eating and lasting for several hours. The pain does not appear to be associated with any particular food. She denies any odynophagia or dysphagia. Ms Smith is a smoker, with a 30-pack year history. You are concerned about peptic ulcer disease. Which additional history finding would increase your level of concern? Choose the single best answer. A. Report of hoarseness B. Epigastic discomfort associated with recumbency or bending C. Bouts of recurrent laryngitis D. Return symptoms despite consistent use of antacids E. Sour or bitter taste in mouth

The correct answer is D. Symptoms that would increase the likelihood of PUD include pain that improves with meals (although some people experience the opposite), history of NSAID use, a pain described as "gnawing" or "hunger-like" and persistent symptoms despite adequate acid-blocking therapy. Hoarseness, epigastric discomfort with lying down, laryngitis, discomfort after large meals, and bitter taste are all typical symptoms of GERD.

Sally is a generally healthy 27-year-old female graduate student who presented to your office twelve weeks ago with episodic post-prandial epigastric burning. This had been bothering her for nearly six months but she had been busy with her thesis and was unable to find the time necessary for an appointment. She reported this year has been particularly stressful, with limited time resulting in increased consumption of coffee and take-out fast food. At that time, she began an eight-week trial of omeprazole. She returns now with no improvement of her symptoms. She discontinued the omeprazole one month ago because she ran out of the medication. She has no additional symptoms and physical exam is unremarkable. Which of the following is the most appropriate next step in her management? Choose the single best answer. A. Refer her for an upper gastrointestinal endoscopy B. Begin treatment with triple therapy of pantoprazole, clarithromycin, and azithromycin for 14 days C. Continue omeprazole for another 4 weeks with close follow up D. Administer a urea breath test E. Switch PPI from omeprazole to pantoprazole

The correct answer is D. This patient exhibits no alarm symptoms, but does have persistent symptoms despite adequate empiric therapy. This case warrants testing for H. pylori as a cause of symptoms. One test which is sensitive and specific for H. pylori infection is the urea breath test. The antibiotic regimen listed in choice B is for treatment of H. pylori, but a diagnosis must be made before instituting such a treatment regimen. Switching PPI's or doing a longer treatment of the same medication would not be optimal choices, since the patient hasn't found any relief thus far with the medication. While an upper endoscopy might assist with diagnosis, it is more invasive and in the absence of alarm symptoms, a less invasive test for H. pylori (urea breath test) is acceptable.

A 57-year-old Caucasian woman comes in, concerned about a 1.5 cm dark multicolored mole on her chin that has been increasing in size over the past 6 months. An excisional biopsy shows pathology indicative of squamous cell carcinoma with clear margin. What is a preventative measure that the patient could implement to prevent further recurrences? A. Use of sunscreen with an SPF of at least 15 B. Avoid artificial sources of UV light, such as indoor tanning C. Visit the doctor every six months for a whole body skin examination D. Wear a wide brimmed hat when in direct sunlight E. All of the above

The correct answer is E. All of the above are important measures to take for skin cancer prevention in someone who has already been diagnosed with one cancerous lesion. All of these measures are appropriate preventative measures for those who have no skin cancer as well, except the frequency of screening exams would be annually instead.

A 58-year-old male presents to clinic with right upper quadrant pain. The patient has a history of hypertension and GERD, but denies any past surgeries. His abdominal exam reveals a positive Murphy's sign. What would be the next best step to confirm your working diagnosis? A. Complete blood count B. Liver function tests C. CT abdomen without contrast D. CT abdomen with contrast E. Abdominal ultrasound

The correct answer is E. Real-time ultrasonography is the preferred study to evaluate right upper abdominal quadrant because it is inexpensive, noninvasive and widely available. US provides good evaluation of gallbladder and is accurate in detection of gallstones as well as dilation of biliary tree. A CT with contrast may give you some evidence of gallbladder disease but is not the preferred test when suspecting biliary colic or cholecystits. A CT without contrast is used primarily when there is a suspicion for kidney stones. The other tests would help with your diagnosis, but US is preferred method for biliary colic/gallstones.

A 35-year-old male presents to clinic with acute onset of right upper quadrant abdominal pain. He denies any past abdominal surgeries or chronic diseases and his only medication is a multivitamin. Which question is most important to include in the review of systems for abdominal pain? A. Have you noticed any weakness or fatigue? B. Have you ever been diagnosed with a hernia? C. Have you ever been diagnosed with a sexually transmitted disease? D. Have you experienced excessive thirst or hunger? E. Have you experienced any dysuria?

The correct answer is E. The systems to include in a review of systems for right upper quadrant abdominal pain are renal, gastrointestinal, pneumonia/pulmonary origins, dermatology, musculoskeletal and neoplastic. In the renal ROS, be sure to illicit any changes in urinary habits like dysuria, polyuria, nocturia or hematuria. In the GI system, ask about history relating to diarrhea, constipation, changes in stool color, blood in stools, nausea and vomiting. To rule out pulmonary origins or RUQ pain, ask about cough, shortness of breath and chest pain. The history relating to dermatologic origins is any new skin rashes to rule out herpes zoster. Musculoskeletal system should be reviewed to illicit any recent trauma and neoplastic processes should be ruled out by asking about fever or weight loss. Options A-D in this question are nonspecific or would not present with pain in the area described in the question.

A 52-year-old woman with a history of diabetes and rheumatoid arthritis presents for her annual examination. She works in an office 10 hours a day, and rarely gets exercise. Her BMI is 23 and her blood pressure is 152/85. Her previous visit two months ago showed blood pressure of 148/82. Her father had a history of diabetes and her maternal grandmother died of rheumatic heart disease at the age of 42. She admits to marijuana drug use in the past and is a nonsmoker. Which of the following is a risk factor for coronary heart disease (CHD) that this patient has? A. Age B. Family history C. Rheumatoid arthritis D. Obesity E. Lifestyle

The correct answer is E. This patient's lack of exercise and sedentary job are risk factors. Age becomes a risk factor over age 55 for women and 45 for men. The family history becomes a risk factor if a first degree relative has CHD male <55 and female <65. Rheumatoid arthritis is not a risk factor, and the patient is not obese.

Mr Gill is a 27-year-old male who presents to his primary care physician with post-prandial epigastric burning. The burning is episodic, without associated hematemesis, dysphagia, or odynophagia. In the past, he has episodically used an OTC chewable calcium carbonate to provide relief. Over the past month, he has had to increase the frequency of its use to four times daily. His blood pressure is 120/81 mm Hg, heart rate at 75/min and regular. Physical exam reveals only minimal epigastric tenderness on palpation but is otherwise normal. Which of the following is the most appropriate next step in management? Choose the single best answer. A. Upper endiscopy B. H. pylori IgG serologic testing C. Fecal immunochemical testing (FIT) D. Fecal occult blood testing (FOBT) E. 8 week trial of pantoprazole

The correct answer is E. This patient displays no red flag symptoms at this time. The most widely accepted initial intervention in a patient like this is empiric treatment with a histamine-2 receptor blocker or a proton pump inhibitor such as pantoprazole. Upper endoscopy would be indicated if there were alarm symptoms or if empiric therapy did not resolve symptoms. H. pylori testing might be indicated if symptoms persisted after empiric therapy. There is not a specific concern for rectal bleeding at this time, so fecal testing for blood is not indicated.

A 68-year-old male with COPD requiring 2L of oxygen at nighttime presents to clinic complaining of increasing lower extremity edema over the past few weeks. He also thinks his nighttime cough might be worse. The mechanism of disease behind the most likely complication is: A. Decreased blood flow to the lower extremities due to thromboembolism B. Hepatomegaly from infiltration of the liver with granulomas C. Irregular heart rate due to atrial fibrillation D. Overexpansion of lower extremity veins due to incompetent venous valves E. Pulmonary hypertension causing right heart failure

The correct answer is E; pulmonary hypertension causing right heart failure. Chronic hypoxia causes pulmonary vasoconstriction that increases blood pressure in the pulmonary vessels. This elevation in blood pressure causes permanent damage to the vessel walls and leads to irreversible hypertension. The right heart eventually fails because the pump cannot sustain flow effectively against this pressure. Right heart failure leads to an increase in preload, with peripheral edema and increased jugular venous distention


Conjuntos de estudio relacionados

ch. 8 practice questions Valuation using the income approach

View Set

Driver Education Chapter 13: Light & Weather Conditions

View Set

CH3: Understanding Cloud Computing QUIZ

View Set

lista de verbos que empiezan con la letra h

View Set

A&P II Ch.22: Immune System and the Body's Defense

View Set

Abeka 7th Grade Science, Section 6.6 Review

View Set